Vous êtes sur la page 1sur 103

MESTRADO PROFISSIONAL EM MATEMTICA EM REDE

NACIONAL - PROFMAT/SBM

OLIMPADA DE MATEMTICA: QUE PRECIOSIDADES ENVOLVEM OS PROBLEMAS


DESTA COMPETIO E QUAL O SEU IMPACTO PARA O PROFESSOR DE
MATEMTICA SEM EXPERINCIA EM OLIMPADAS E A SUA IMPORTNCIA PARA O
ESTUDANTE?

Autor : Carlos Alberto da Silva Victor


Orientador : Wanderson Jos Lambert

UFRRJ-2013

AGRADECIMENTOS

Inicialmente agradeo a todos os organizadores deste grande projeto PROFMAT, que


possivelmente enfrentaram infinitas dificuldades, que certamente desconhecemos, para que
este intento em rede nacional tivesse o sucesso alcanado. Sou grato a todos pela alegria que
ns, professores de Escola Pblica Bsica do Brasil, estamos sentindo pela oportunidade
mpar de nos aperfeioarmos.
Agradeo ao meu orientador Wanderson Jos Lambert, e tambm professor de trs
disciplinas do nosso curso, cujo carisma e pacincia divina, fizeram com que essa dissertao
tivesse a coerncia de toda a minha proposta de trabalho. Ao mestre Orlando dos Santos
Pereira, coordenador do PROFMAT-UFRRJ e a todos os mestres das disciplinas, o meu
muito obrigado pelas orientaes e grandes discusses obtidas durante todo o curso.
Agradeo tambm ,ao meu mestre Pedro Carlos Pereira, pelo considerao e confiana
surgidas em todas as discusses que tivemos em sala de aula, na disciplina de Histria da
Matemtica.

Daisy

Aparecida Nogueira, os meus sinceros agradecimentos pela sua reviso

ortogrfica e a sua incansvel boa vontade em tornar este trabalho consistente.


Aos meus colegas de turma, obrigado pelos momentos inesquecveis que passamos juntos;
certamente, eles ficaro em minha memria por toda a vida daqui para a frente.
A Luiz Amorim Goulart, um agradecimento especial, pelo constante incentivo para que este e
outros trabalhos fossem realizados.
A Rafael Castiglione por suas orientaes computacionais infalveis nos momentos em que
as brigas com os arquivos instalavam-se.
A Antonio Bartolomeu pela confiana platnica e sua pacincia indiscritvel em todos os
momentos difceis.
Antonio Luiz Santos (Gandhi), pelas valiosas contribuies durante anos, em mostrar
muitos caminhos descritos neste trabalho e por estar sempre presente para qualquer tipo de
ajuda.
Finalmente a todos aqueles que diretamente ou indiretamente, estiveram junto comigo nesta
trajetria.

DEDICATRIA

A meu pai, Francisco, por sua orientao durante toda a minha caminhada.
minha me, Maria Dinorah, por sua infinita preocupao pelos caminhos que segui.
minha esposa, Maria de Lourdes, por sua fora incomparvel de luta e compreenso, para
que tudo isso fosse realizado.
Em Memria do meu irmo, Agenor, por alegres momentos que passamos juntos.
E, especialmente, em Memria minha cachorrinha, Mel, companheira fiel que
compartilhou, por 12 anos, as minhas tristezas e as minhas alegrias.

RESUMO
VICTOR, Carlos Alberto da Silva. Olimpada de Matemtica: que preciosidades
matemticas envolvem os problemas desta competio e qual o seu impacto para o
professor de matemtica sem experincia em olimpadas e a sua importncia para o
estudante? 95 p., Dissertao (Mestrado em Rede Nacional PROFMAT). Instituto de
Cincias Exatas, Departamento de Matemtica, Universidade Federal Rural do Rio de Janeiro,
RJ, 2013.

A Olimpada de Matemtica na busca de novos talentos tem a caracterstica de ser uma


competio intelectual, utilizando para isto problemas desafiadores que exigem do aluno a sua
capacidade criativa na resoluo dos mesmos. Em geral, se no houver uma preparao
especfica, deparamo-nos com vrias barreiras que esto escondidas nas teorias. Este fato
pode ser comum tanto ao discente quanto ao docente e, principalmente, nas fases finais das
Olimpadas. Um sentimento de frustrao nos abrange quando no sabemos de que forma
devemos enfrentar tais barreiras. Temos aqui uma proposta na tentativa de diminuir a
distncia que ocorre com matemtica nas Escolas Bsicas e a Olimpada de Matemtica. A
dinmica dessa proposta foi baseada em experincias no preparo de alunos para as Olimpadas
e de alguma forma tentando mostrar como podemos crescer teoricamente atravs dos
problemas envolvidos. Buscamos atravs da resoluo de alguns problemas escolhidos traar
algumas estratgias interessantes para a resoluo dos mesmos utilizando pouca teoria. Em
tais resolues fazemos comentrios dessas estratgias e de possveis dificuldades que o aluno
e/ou professor possam se deparar. Esta experincia indicou que h vrios problemas que
estimulam os jovens a gostar de matemtica e se interessar pelos Problemas Olmpicos,
bastando para isto desenvolver o raciocnio dele atravs de assuntos que possuem facilidades
na aprendizagem. Como complementao, fazemos uma discusso no exaustiva de vrios
livros ou textos utilizados nesta experincia para o preparo de alunos s diversas competies
Olmpicas dentro da Matemtica.

Palavras-chave: Problemas de Olimpadas de Matemtica, Matemtica na Escola Bsica,


Preparao de alunos e professores, Descrio de Livros para preparao em Olimpadas.

ABSTRACT

VICTOR, Carlos Alberto da Silva. Math Olympiad: that mathematical morsels involve
the problems of this competition and its impact to the math teacher with no experience
in the Mathematical Olympiad and its importance to the student? 95 p., Thesis (MA in
National Network - PROFMAT). Institute of Mathematical Sciences, Department of
Mathematics, Federal Rural University of Rio de Janeiro, RJ, 2013.

The Mathematical Olympiad in searching of new talent has the characteristic of being an
intellectual competition, using for this challenging problems which require from the student
his creative ability in solving them. In general, if there is no specific preparation, we are faced
with several barriers that are hidden in the theories. This may be common to both student as
the teacher, and especially in the final stages of the Mathematical Olympiad. A feeling of
frustration covers us when we do not know how we should address such barriers. Here we
have a proposal in an attempt to bridge the gap that occurs with mathematics in basic schools
and Mathematical Olympiad. The dynamics of this proposal was based on experiences in
preparing students for the Mathematical Olympiad and somehow trying to show how we can
theoretically grow through the issues involved. We seek through solving some problems
chosen to draw some interesting strategies for solving them by using little theory. In such
resolutions we do reviews on these strategies and possible difficulties that the student and / or
teacher may find. This experience indicated that there are several problems that encourage
young people to enjoy and take an interest in solving problems enough for this to develop his
reasoning through issues that have abilities for learning. Summarizing, we complement our
discussion by analysing several books and texts used in this experience to prepare students
for various competitions within the Mathematical Olympiad.

Keywords: Mathematical Olympiad Problems, Mathematics in Primary School, Preparing


students and teachers, Description of books in preparation for the Mathematical Olympiad.

NDICE DOS PROBLEMAS


PROBLEMA 1( quadrado dividido em 1993 outros quadradinhos) .......................................... 9
PROBLEMA 2 ( quadrado dividido em noutros quadradinhos)........................................... 9
PROBLEMA 3 (soma de inversos de naturais) .......................................................................... 9
PROBLEMA 4 (uma fatorao com conseqncias fascinantes) ............................................ 10
PROBLEMA 5 ( de novo a fatorao do problema 4) ............................................................. 10
PROBLEMA 6 ( que incrvel princpio!) .................................................................................. 11
PROBLEMA 7 ( de tirar o flego ! )...................................................................................... 11
PROBLEMA 8 ( uma soma que d o que falar) ....................................................................... 11
PROBLEMA 9 ( sabe que eu pensei em multiplicar tudo por x?) ....................................... 12
PROBLEMA 10 ( rapaz ! o que que eu fao aqui ?) ................................................................ 12
PROBLEMA 11 ( ainda no dei esta matria !! ) .................................................................... 12
PROBLEMA 12 ( estou s olhando pr ela !) .......................................................................... 13
PROBLEMA 13 ( preciso resolver mais problemas de polinmios !!) ................................. 13
PROBLEMA 14 ( est me dando raiva !!)................................................................................ 13
PROBLEMA 15 ( por que que no meu livro no tem isto ? e agora ?)................................... 13
PROBLEMA 16 ( deste tipo eu no sabia!).............................................................................. 14
PROBLEMA 17 ( a voc est querendo me derrubar!!) ........................................................ 14
PROBLEMA 18 ( s sabia fazer por produtos notveis!) ........................................................ 14
PROBLEMA 19 ( Eu no sei esta tal de Diofantina ! ) ............................................................ 14
PROBLEMA 20 ( rapaz, eu pensei substituir valores !) .......................................................... 15
PROBLEMA 21 ( por geometria ? deve ter um traado mgico!) ........................................... 15
PROBLEMA 22 ( muita imaginao!) ................................................................................... 15
PROBLEMA 23 ( instigante hein ! ) ......................................................................................... 15
PROBLEMA 24 ( at d pra pensar em alguma coisa!) .......................................................... 16
PROBLEMA 25 ( deve ser desigualdade triangular, eu acho!) ............................................... 16
PROBLEMA 26 ( vou sair no brao!) ...................................................................................... 16
PROBLEMA 27 ( esta de alguma forma sai!) .......................................................................... 16
PROBLEMA 28 ( nossa! O que isto meu camarada ?) ........................................................ 17
PROBLEMA 29 ( esta acho que tambm sai !) ........................................................................ 17
PROBLEMA 30 ( pr que esta complicao!) ......................................................................... 17
PROBLEMA 31 ( sei l o que eu vou fazer!)............................................................................ 17

PROBLEMA 32 ( isto t me cheirando mdias!) ..................................................................... 18


PROBLEMA 33 ( esses caras ficam inventando!) .................................................................... 18
PROBLEMA 34 ( por onde eu comeo ?)................................................................................. 18
PROBLEMA 35 ( acho que vou usar a calculadora!) .............................................................. 18
PROBLEMA 36 ( essas desigualdades me deixa pasmo!) ...................................................... 18
PROBLEMA 37 ( vou traar paralelas ao lados do quadrado a partir de P!) ........................ 19
PROBLEMA 38 ( esta no d pra mim!)................................................................................. 19
PROBLEMA 39 ( hum! 121 = 11x11 !) .................................................................................... 19
PROBLEMA 40 ( rapaz , voc tem imaginao!) .................................................................... 20
PROBLEMA 41 ( a primeira eu fao no brao mesmo, mas a segunda , no sei no!) ....... 20
PROBLEMA 42 ( saindo no brao,acho que d!) .................................................................... 20
PROBLEMA 43 ( caramba!) .................................................................................................... 20
PROBLEMA 44 ( isto me assusta!) .......................................................................................... 20
PROBLEMA 45 ( eu vou desenvolver tudo e....!) ..................................................................... 21
PROBLEMA 46 ( isto loucura , vou particularizar!) ......................................................... 21
PROBLEMA 47 ( o meu camarada! Por que voc faz isto comigo!) ....................................... 21
PROBLEMA 48 ( vou sair no brao tambm e vejo o que d no final!) .................................. 21
PROBLEMA 49 ( sem dizer a quantidade de dgitos !)............................................................ 22
PROBLEMA 50 ( este o mesmo que ganhar na MegaSena!) ............................................... 22

NDICE DAS SOLUES


Soluo do Problema 1 ............................................................................................................ 23
Soluo do Problema 2 ............................................................................................................ 24
Soluo do Problema 3 ............................................................................................................ 26
Soluo do Problema 4 ............................................................................................................ 27
Soluo do Problema 5 ............................................................................................................ 29
Soluo do Problema 6 ............................................................................................................ 30
Soluo do Problema 7 ............................................................................................................ 31
Soluo do Problema 8 ............................................................................................................ 33
Soluo do Problema 9 ............................................................................................................ 34
Soluo do Problema 10 .......................................................................................................... 35
Soluo do Problema 11 .......................................................................................................... 37
Soluo do Problema 12 .......................................................................................................... 38
Soluo do Problema 13 .......................................................................................................... 39
Soluo do Problema 14 .......................................................................................................... 40
Soluo do Problema 15 .......................................................................................................... 40
Soluo do Problema 16 .......................................................................................................... 42
Soluo do Problema 17 .......................................................................................................... 42
Soluo do Problema 18 .......................................................................................................... 43
Soluo do Problema 19 .......................................................................................................... 44
Soluo do Problema 20 .......................................................................................................... 44
Soluo do Problema 21 .......................................................................................................... 45
Soluo do Problema 22 .......................................................................................................... 46
Soluo do Problema 23 .......................................................................................................... 47
Soluo do Problema 24 .......................................................................................................... 48
Soluo do Problema 25 .......................................................................................................... 49
Soluo do Problema 26 .......................................................................................................... 50
Soluo do Problema 27 .......................................................................................................... 52
Soluo do Problema 28 .......................................................................................................... 52
Soluo do Problema 29 .......................................................................................................... 53
Soluo do Problema 30 .......................................................................................................... 53
Soluo do Problema 31 .......................................................................................................... 54
Soluo do Problema 32 .......................................................................................................... 54

Soluo do Problema 33 .......................................................................................................... 55


Soluo do Problema 34 .......................................................................................................... 56
Soluo do Problema 35 .......................................................................................................... 57
Soluo do Problema 36 .......................................................................................................... 58
Soluo do Problema 37 .......................................................................................................... 58
Soluo do Problema 38 .......................................................................................................... 60
Soluo do Problema 39 .......................................................................................................... 61
Soluo do Problema 40 .......................................................................................................... 61
Soluo do Problema 41 .......................................................................................................... 61
Soluo do Problema 42 .......................................................................................................... 63
Soluo do Problema 43 .......................................................................................................... 63
Soluo do Problema 44 .......................................................................................................... 63
Soluo do Problema 45 .......................................................................................................... 64
Soluo do Problema 46 .......................................................................................................... 65
Soluo do Problema 47 .......................................................................................................... 65
Soluo do Problema 48 .......................................................................................................... 66
Soluo do Problema 49 .......................................................................................................... 67
Soluo do Problema 50 .......................................................................................................... 67

NDICE DAS TABELAS


Tabela 1(quantidade mnima para n = 2) ................................................................................ 30
Tabela 2(quantidade mnima para n = 3) ................................................................................ 30

NDICE DAS FIGURAS

Figura 1 (Ponto interno no quadrado) ..................................................................................... 19


Figuras 2 e 3 ( diviso do quadrado em 4 e 7 quadradinhos ) ................................................ 23
Figura 4 e 5 ( diviso do quadrado em 6 e 9 quadradinhos) ................................................... 25
Figura 6 e 7 ( diviso do quadrado em 8 e 11 quadradinhos) ................................................. 25
Figura 8 ( um antigo problema que nos deixa perplexos) ....................................................... 31
Figura 9 ( que genialidade!) .................................................................................................... 32
Figura 10 ( verificando se h soluo) .................................................................................... 38
Figura 11 ( um circuncentro e tanto ! ) .................................................................................... 45
Figura 12 ( uma soluo geomtrica divina) ........................................................................... 46
Figura 13 ( um quadriltero que no se esquece) ................................................................... 49
Figura 14 ( muito interessante ) ............................................................................................... 50
Figura 15 ( que ponto particular!) ........................................................................................... 51
Figura 16 ( muita observao!) ............................................................................................... 51
Figura 17 ( uma reta abenoada ) ........................................................................................... 55
Figura 18 ( qual a ideia?)...................................................................................................... 59
Figura 19 ( s poderia ser de Murray Klamkin ) ..................................................................... 59
Figura 20 ( s Pitgoras ) ........................................................................................................ 66

SUMRIO
INTRODUO .. .......................................................................................................................1
MANUAL DO PROFESSOR.......................................................................................................7
CAPTULO 1 : ENUNCIADOS DOS PROBLEMAS .............................................................. .9
CAPTULO 2 : SOLUES DOS PROBLEMAS ................................................................. 23
CAPTULO 3 : DESCRIO DE LIVROS, REVISTAS E ARTIGOS PARA AS OLIMPADAS
DE MATEMTICA...........................................................................................67
CONSIDERAES FINAIS ................................................................................................. 80
REFERNCIAS . ....................................................................................................................83
APNDICES ........................ ................................................................................................ 85
APNDICE A : SEMANA OLMPICA ................................................................................. 85
APNDICE B : RETA DE SIMSON ..................................................................................... 86
APNDICE C : POTI - POLOS OLMPICOS DE TREINAMENTO INTENSIVO ................. 87
APNDICE D : NGULOS ADVENTCIOS............................................................................88
APNDICE E : SOLUO TRIGONOMTRICA PARA O
PROBLEMA7............................................................................................................................89

APNDICE F : SOLUO TRIGONOMTRICA PARA O


PROBLEMA21..........................................................................................................................90

INTRODUO
Este trabalho teve sua motivao inicial atravs de uma experincia prpria com
relao s Olimpadas de Matemtica na dcada de 90. Mesmo com grande experincia em
preparatrios para os diversos vestibulares militares e civis, a dificuldade em resolver
determinadas questes das Olimpadas de Matemtica se tornava evidente. O porqu de tais
dificuldades era uma reflexo comum tambm a vrios colegas de profisso quando se
deparavam com essas provas, principalmente nas fases finais da competio.
O que mais incomodava era que as dificuldades se tornavam cada vez maiores, mesmo
tendo cincia de que os problemas envolvidos, em sua grande maioria, eram e so atualmente
de contedos da grade curricular da Matemtica do Ensino Bsico.
Apesar de a dificuldade ser um conceito relativo, existem professores de Matemtica
de diversos nveis de ensino que se deparam com inmeros obstculos quando tentam
solucionar algumas questes dessas Olimpadas, e isso pode no traduzir o conhecimento
matemtico deles em termos de contedos do currculo dos Ensinos Bsico ou Superior, j
que a abordagem das questes tem um formato que muitas vezes difere do que estamos
acostumados a encontrar nos livros textos, o que talvez atrapalhe a compreenso ou o
desenvolvimento de tais problemas.
As questes propostas nas Olimpadas de Matemtica so, em geral, desafiadoras,
instigantes, renovadoras e, como a competio tem um carter intelectual, as resolues
exigem do candidato a capacidade de abstrao, criatividade e um raciocnio que, em geral,
depende, de um treinamento. Os prprios stios na internet das Olimpadas destacam que um
dos objetivos estimular o jovem a estudar Matemtica, alm da busca de novos talentos.
O prprio projeto de Olimpadas de Matemtica descreve a importncia de termos no
Brasil uma competio de carter nacional:
Temos hoje brilhantes matemticos e cientistas de renome mundial
que tiveram origem nas Olimpadas de Matemtica. Entretanto,
reconhecemos que, com esta atividade, pode-se fazer muito mais.
Com parceria do IMPA (Instituto de Matemtica Pura e Aplicada) e
com a SBM (Sociedade Brasileira de Matemtica), foi submetido ao
CNPq um projeto que pretende contribuir para a melhoria do ensino
de Matemtica no Brasil utilizando as Olimpadas de Matemtica

como mecanismo propagador. (revista Eureka! nmero 1-1998apresentao).


Para que esse jovem seja estimulado a participar de tais competies ou mesmo para
que ele tenha o interesse em estudar Matemtica, existe a necessidade da participao
intensiva do professor, o qual deve ter a vontade de desenvolver em suas aulas, ou em
atividades extraclasse, questes inerentes a esse tipo de competio. Como consequncia, o
docente se v obrigado a buscar um aperfeioamento e, com isso, a melhoria no ensino de
Matemtica nas Escolas Bsicas.
Um questionamento ocorre em funo do pargrafo anterior: De que forma deve o
professor se aperfeioar?. Somente com as solues das provas anteriores, talvez no haja
material suficiente para capacitar o professor, a fim de que ele prepare os alunos para as
Olimpadas de Matemtica. Conforme Biondi,Vasconcelos e Menezes [4] explicam, o Banco
de Questes (veja a Seo 3.1) ajuda na melhoria da qualidade do desempenho dos alunos
nas avaliaes da Olimpada Brasileira de Matemtica das Escolas Pblicas (OBMEP).
Entretanto, pelo que se pode observar, muitas vezes, o professor e aluno no so instigados a
pensar ou seguir os passos que levaram o autor a chegar soluo.
Acreditamos, tambm, que em algumas questes, como, por exemplo, do Ensino
Mdio, conhecer determinados contedos, de Teoria dos Nmeros, Anlise, lgebra,
Combinatria, Geometria, Trigonometria, entre outros, tornam mais seguros o raciocnio e a
criatividade, o que nos conduzir a um preparo mais amplo em relao ao currculo da
Matemtica Elementar; alm disso, ter uma bibliografia que desenvolva essas habilidades
um fato que devemos levar em considerao.
A revista Eureka! de 98 ( nmero 1) tratou do tema; segundo ela:
A cada ano, livros novos so editados
repetindo quase sempre o mesmo estilo e os mesmos contedos dos
anteriores. Existem hoje no Brasil bons livros de Matemtica
dedicados aos alunos tanto do ensino fundamental quanto do ensino
mdio. Entretanto, o que lhes falta um ingrediente que, no mundo
de hoje, fundamental: o estmulo criatividade. Entendemos que
no suficiente para a formao do futuro cidado um aprendizado
burocrtico da Matemtica e percebemos a importncia de estimular
os alunos desde tenra idade a resolver problemas novos e

desafiantes, propiciando o desenvolvimento da imaginao e da


criatividade.
O

programa

de

Olimpadas

de

Matemtica

reconhecido em todos os pases do mundo desenvolvido como o mais


eficiente instrumento para atingir esse objetivo. Aproveitando o
natural gosto dos jovens pelas competies, as Olimpadas de
Matemtica tm conseguido estimular alunos a estudar contedos
alm do currculo escolar e, tambm, por outro lado, aumentar e
desenvolver a competncia dos professores. (EUREKA!-1998).
O incio em 2011 do programa de Mestrado Profissional em Matemtica em Rede
Nacional (PROFMAT) trouxe uma oportunidade para o professor que atua na Educao
Pblica de um aprimoramento na sua formao. Os contedos idealizados pelos
coordenadores e inicialmente disponibilizados no stio do PROFMAT para os participantes
do mestrado foram editados no ano de 2012 em livros da Coleo PROFMAT. A natureza
dessa coleo tem um aprofundamento e uma nova viso dos assuntos ministrados nas escolas
bsicas do Brasil e, certamente, so um grande apoio e incentivo para o professor na
qualidade de sentir-se mais seguro em sala de aula.
O que notamos com relao a esse programa de mestrado PROFMAT , que sua
coleo, muitas vezes nos desenvolvimentos dos contedos e dos exerccios propostos, tem
uma conexo com o programa de Olimpadas de Matemtica no Brasil, como, por exemplo, a
disciplina de Resolues de Problemas, obrigatria no mestrado.
Acreditamos que dessa forma o professor ser capaz de criar estratgias em suas
escolas juntamente com os orientadores, diretores e a Secretaria de Educao, como, por
exemplo, competies na sua prpria escola ou com outras, de modo que um aluno, que por
algum motivo no goste de Matemtica, venha a se interessar a estudar a disciplina,
participando das Olimpadas no por obrigao, mas por vontade prpria. Para os alunos que
gostam, surgiro oportunidades de mostrar os seus potenciais e, quem sabe, vir a descoberta
de novos talentos. Entendemos, tambm, que todo esse processo de aprendizagem mais uma
forma de integrao social do cidado, visto que ele poder encontrar jovens da mesma faixa
etria com objetivos semelhantes .
Ressaltamos que a prtica de resolues de problemas nos fortalece e nos faz sentir
mais seguros na busca de novos conhecimentos tornando-se muito gratificante quando, aps
vrias tentativas, a soluo surge e, quebrando o desafio descrito na questo.
3

O problema 21 do captulo 2, cujo enunciado consta de uma revista Australiana


denominada Function A School Mathematics Journal - Volume 21 Part 5 October 1997Problem 21.5.2, pgina 167, da seo de problemas, um dos exemplos da incessante procura
pessoal para uma soluo geomtrica. Durante aproximadamente 30 dias pensando nesse
problema, em que uma possvel frustrao j se instalava pela no resoluo da questo, o
esperado ocorre como um acaso aps um determinado traado. Situao semelhante,
provavelmente, j ocorreu com vrios professores; a soluo se evidencia, e a perplexidade
nos questiona: o que nos levou a realizar esse traado? Tal fato vem a confirmar que tentar
requer pacincia (vide cap 3 tem 3.2-- introduo). Transferir essa satisfao para o aluno
, tambm, um dos nossos desafios.
H dissertaes e trabalhos que descrevem as origens das Olimpadas de
Matemtica, nos quais so analisados os objetivos, os regulamentos, suas conseqncias na
qualificao do ensino da disciplina na Educao Bsica e os impactos para os alunos das
escolas pblicas.
Um exemplo desses trabalhos pode ser encontrado em uma dissertao de mestrado
[1], na qual o autor discute, com aprofundamento, tais origens e as diversas Olimpadas de
Matemtica, inclusive Internacionais. Em [1], o autor cita trabalhos correlatos de Nascimento
e Oeiras [12] e um estudo de Sucupira [19] sobre a participao feminina nas Olimpadas de
Matemtica, atravs de contribuies dos membros do Conselho de Administrao do IMPA
de Bondi, Vasconcelos e Menezes [4] e Peraino [13]. O autor descreve, atravs de
questionrio, uma pesquisa entre professores e alunos, e como o comportamento do aluno
em participar nesse tipo de competio .
Outro trabalho interessante sobre o assunto [10], no qual descrita em linhas gerais
a OBMEP, sua origem e seus possveis impactos no ensino de Matemtica na Escola Pblica
no Brasil. Todos esses trabalhos indicados nas referncias bibliogrficas descrevem, tambm
as origens e os regulamentos das Olimpadas Internacionais.
Entretanto, h poucos trabalhos discutindo quais os caminhos a seguir na resoluo
dos problemas, como enfrentar os obstculos nas questes propostas e obteno de
motivaes para outros problemas.
A meta desse trabalho tentar amenizar dificuldades semelhantes s que surgiram
quando nos debruamos sobre as questes e, apresentamos caminhos aos que pretendem
iniciar-se nessa arte de resolver problemas, objetivando que esse no seja mais um artigo de
solues, mas que, atravs dos problemas aqui sugeridos, nasa um estmulo para o professor
4

ou leitor iniciantes em Olimpadas de Matemtica a fim de que se sintam mais confortveis ao


se depararem com essas provas.
O trabalho est dividido em trs captulos, a saber:
No Captulo 1, os enunciados de 50 problemas so apresentados com o objetivo de que
o leitor tente, inicialmente, resolv-los; neles so descritos origem e finalidade de cada um,
bem como uma sugesto que pode ajudar na soluo.
No Captulo 2, uma possvel soluo para os problemas, alm de comentrios
relevantes ao desenvolvimento da questo. Deixaremos, a partir do problema 33, que o leitor
faa os prprios comentrios com relao aos problemas apresentados, e s solues, para que
a experincia dele em sala de aula no necessariamente para o preparo de alunos para as
Olimpadas de Matemtica possa enriquecer esse trabalho para uma futura verso.
Nesse captulo observaremos como problemas aparentemente complexos para muitos
podem ser resolvidos com ferramentas do Ensino Bsico, utilizando estratgias que
evidentemente no so nicas, mas que, futuramente, possam ser teis para novos problemas.
No captulo 3, apresentamos uma biblioteca que nos conduziu a um aperfeioamento e
nos ajudou no preparo e na orientao dos nossos jovens para a competio. Uma descrio
dos seus contedos tambm mencionada.
Os 50 problemas do captulo 2 foram selecionados visando que o leitor venha a ser
estimulado atravs das solues apresentadas, interessando-se por problemas desafiadores
propostos nas Olimpadas, revistas ou livros que contenham questes nesse estilo.
Destacamos detalhes nos contedos programticos exigidos do ensino da Matemtica
nas escolas de Ensinos Fundamental e Mdio que geralmente no esto presentes nos livros
textos bsicos. Todas as solues apresentadas so, em sua maioria, das bancas examinadoras,
dos livros ou artigos que constam no captulo 3 ou nas referncias bibliogrficas, ou ainda,
fruto de discusses entre grupos de professores do qual fazemos parte, bem como de alunos
participantes da competio.
Destacamos que no foram inseridas questes de determinados contedos, como por
exemplo, Trigonometria, Anlise, reas afins da Aritmtica (como por exemplo;
Congruncias), Jogos Matemticos (os quais trazem bastantes benefcios para a criatividade e
o desenvolvimento do perfil de um aluno olmpico), entre outras, por se tratar de um primeiro
trabalho cuja finalidade levar o iniciante em Olimpadas a pensar com menos teoria
possvel. Tais assuntos podem ser encontrados nos livros do Captulo 3, bem como em outras
literaturas, que certamente faro parte de novas dissertaes ou livros do gnero de nosso
5

trabalho. Importante destacar que existe uma infinidade de problemas que poderiam ser
inseridos no captulo 1 com o mesmo propsito.
Para salientarmos a beleza de uma prova de Olimpada de Matemtica e o que ela
pode proporcionar em termos de originalidade, criatividade e imaginao, fecharemos o
quinquagsimo problema do captulo 1 com um surpreendente resultado que pode deixar
perplexos mesmo aqueles experientes em Olimpadas. claro que temos, em Matemtica,
outros resultados fascinantes, porm, esse foi escolhido pela originalidade e porque um
aluno do 9 ano do Ensino Fundamental consegue entend-lo aps a soluo, alm de esse
aluno ser capaz de criar outros no mesmo estilo.
As figuras presentes neste trabalho foram geradas inicialmente pelo software Cabri
Geometry II Plus e em algumas vezes sendo ajustadas no Paint do Windows ; apenas a Figura
10 foi gerada pelo software GeoGebra.
Esperamos que novos trabalhos do mesmo gnero surjam com o objetivo de propiciar
ao professor de Matemtica ou o leitor interessado, que de alguma forma temeroso ao se
deparar com uma Olimpada momentos de deleite nas resolues e que essas literaturas
sejam estimulantes na prtica de resolver problemas similares aos propostos na competio.

MANUAL DO PROFESSOR
Para melhor perfomance futura e para um conforto em relao s questes de
Olimpadas, relatarei aqui alguns passos pelos quais podemos desenvolver a teoria e
problemas referentes aos contedos do Ensino Bsico. Certamente, aps um tempo em que o
amadurecimento na prtica de problemas torna-se uma realidade, o professor tomar os
prprios rumos para o mundo das Olimpadas de Matemtica e possivelmente uma alterao
enriquecedora deste manual.
O passo inicial que, de alguma forma, dentro do contedo que est sendo estudado
no momento, devemos utilizar alguma estratgia de modo que o aluno se sinta motivado.
Podemos desenvolver um problema com o aluno, conduzindo-o resposta, encaminhando a
resoluo e sugerindo alguns passos. Esse procedimento ser muito gratificante para o
estudante, o qual se sentir fortalecido para solues futuras de outros problemas.
Mesmo que o docente no tenha a prtica de alguns contedos no estudados no
Ensino Bsico, como, por exemplo, congruncias, ele pode idealizar um problema real em que
essa teoria possa ser usada, sendo aproveitada pelo professor para que seja estudada,
juntamente com o aluno. Essa foi uma das estratgias de que lancei mo, e inicialmente
extraclasse. Exemplo : dois sacos de moedas so tais que : o primeiro (saco 1) contm duas
moedas de um real e um outro saco ( saco 2), contm trs moedas de um real . Quantos sacos
de (1) e de (2) devemos ter, para pagar uma conta de 137 reais ?

Passos sugeridos:
(I) Escolha, inicialmente, a rea na qual mais se sinta vontade (Aritmtica, lgebra
ou Geometria); procure desenvolver com os alunos questes um pouco mais sofisticadas .
Faa com eles as que tenham resultados surpreendentes (problema 1 do Cap 1, por exemplo);
isso, certamente, chamar a ateno dos estudantes, e voc se sentir como um agente
transformador e um expectador motivado. Um exemplo para um aluno de 9 ano resolver
uma equao do 2 grau, utilizando apenas a fatorao e completando os quadrados.
(II) Os livros citados em (3.6), (3.7) e (3.8) so fundamentais para todo esse incio.
(III) Estude com os alunos os artigos para iniciantes da revista Eureka!.
(III) Desenvolva com os alunos o Vol 1 da coleo correspondente de (3.9) .
(IV) Siga as informaes do Captulo 3.

Agora, uma dica importante:


Tenha o hbito de resolver problemas, perca tempo pensando, no desista, tenha
pacincia, e logo vers os resultados surpreendentes; busque novos conhecimentos nos
livros descritos neste trabalho. Escolha um dos problemas propostos da revista Eureka!,
fique em cima dele, trabalhe com pacincia. Se a ideia no aparecer logo, pare um pouco
e, assim que tiver um tempinho, pense nele novamente. Se mesmo assim nenhuma ideia
surgir, escolha um outro problema e continue tentando. Afirmo que em breve voc estar
dando solues brilhantes ....

CAPTULO 1 : ENUNCIADOS DOS PROBLEMAS

Neste captulo temos os enunciados dos problemas que na sua maioria fizeram parte
de Olimpadas. Comentrios sobre suas origens e sugestes para uma possvel soluo sero
apontados apenas com o intuito de facilitar as idias. Mas, sugerimos que antes do leitor ir ao
Captulo 2 que tente resolver alguns desses problemas. No se preocupe pelo tempo que
muitos poderiam vir a pensar que perdido com as tentativas, muitos especialistas em
Olimpadas passaram por isto e todo este processo faz parte do desenvolvimento criativo.
No momento que estudar a soluo, verifique a possibilidade de uma outra de modo
que esta tenha um outro formato e talvez uma resoluo com maior brilhantismo. Caso isso
no ocorra, no desanime; com o decorrer do tempo novas idias surgiro e breve ters
sucesso. Viaje nesse valioso mundo dos problemas de Olimpadas de Matemtica. Lembre-se:
tentar requer pacincia.

PROBLEMA 1 ( quadrado dividido em 1993 outros quadradinhos)


possvel dividir um quadrado em 1993 outros quadradinhos ? esses quadradinhos no so
necessariamente congruentes.

Esta questo foi parte da prova da Olimpada de Colorado (1993) [17]. Esboar um
quadrado e dentro deste montar outros quadradinhos menores uma estratgia para a soluo.
PROBLEMA 2 ( quadrado dividido em noutros quadradinhos)
(generalizao do problema 1) possvel dividir um quadrado em n outros quadradinhos?
Esses quadradinhos no so necessariamente congruentes.

No se sabe se o Problema 1 teve a sua origem desta generalizao, pois tal fato no
comentado no livro em que o problema 1 est resolvido.

PROBLEMA 3 (soma de inversos de naturais)


(problema de uma Olimpada Russa) Prove que a soma

1 1
1 2

1
3

1
1
...
4
n

n
i

1
1 i

no um valor inteiro.
Esta questo foi proposta numa Olimpada Russa (vide [16]) e cuja soluo origina
uma propriedade interessante descrita a seguir.
Tente inicialmente pensar na maior potncia de 2 de 1 at n e logo aps faa o M.M.C
dos denominadores.

PROPRIEDADE 1 : Todo nmero natural diferente de zero , pode ser escrito na forma 2k .I ,
onde k um inteiro no negativo e I um mpar .

Observe que esta propriedade aparece exatamente quando trabalhamos com fatorao
em nmeros primos. Veja os exemplos :

17 20.17;
22 21.11.

PROBLEMA 4 (uma fatorao com conseqncias fascinantes)


Considere um paraleleppedo retngulo de lados variveis e reais tais que o permetro
constante igual a P . Determine o volume mximo deste paraleleppedo e suas dimenses .

uma questo que em geral aparece em livros de Clculo diferencial e integral e em


artigos de desigualdades. A soluo apresentada utilizando uma fatorao de
a3 b3 c3 3abc

no muito comum em livros textos do ensino bsico. Utilizaremos tambm uma

propriedade aparentemente simples, mas que resolve muitos problemas.

PROPRIEDADE 2: O quadrado de qualquer nmero real sempre maior do que zero ou


igual a zero.

PROBLEMA 5 ( de novo a fatorao do problema 4)


Suponha que tenhamos a,b,c nmeros reais no nulos , tais que a b c 0 . Determine o
valor da expresso

a2
bc

b2
ac

c2
.
ab

10

Esta questo foi inspirada numa outra proposta no livro de Titu Andreescu [2].

PROBLEMA 6 ( que incrvel princpio!)


Qual a quantidade mnima k de pessoas presentes numa reunio, para que possamos
garantir que n pessoas nasceram no mesmo dia da semana ?
Questo proposta para que o aluno perceba que atravs de uma situao do dia a
dia, na verdade, est diante de um princpio que , alm de resolver vrios problemas de
olimpadas, garante uma aplicao.

PROBLEMA 7 ( de tirar o flego ! )


Seja ABC um tringulo tal que AB=AC e que

BAC

200 . Sobre os lados AB e AC,

marcamos os pontos P e Q respectivamente, tais que


Determine

PCB 6O e CBQ 5O .

CPQ .

Esta questo uma das propostas no livro Problemas de Geometria Planimetria de


I.Shariguin , editora Mir Mosc e se trata de problemas denominados de ngulos
adventcios, que so ngulos nos quais so resultados dados por nmeros inteiros, quando
se faz traados como na figura desse problema ( veja o Apndice D).
uma questo que se faz presente em algumas apostilas e livros de

cursos

preparatrios e, possui um grau alto de dificuldade pois somente com a Lei Angular de
Tales, a soluo se torna impossvel.

PROBLEMA 8 ( uma soma que d o que falar)

Prove que x

1
x

2 ou x

1
x

2,x

0 e real.

Essas duas desigualdades foram mencionadas devido a duas propriedades dos nmeros
reais em que geralmente no constam de livros textos bsicos tanto do ensino fundamental e
mdio. Geralmente, quando elas so apresentadas aos alunos, h uma reao de surpresa pelo
fato de ser fcil a verificao numrica e eles nunca terem visto nas aulas de matemtica.
Constataremos durante os problemas como essas desigualdades nos ajudam!.
11

PROBLEMA 9 ( sabe que eu pensei em multiplicar tudo por x?)


13

Resolva nos reais a equao :

13

x12
x

13

Questo semelhante foi alvo do concurso para os alunos do Colgio Naval .

Sugesto : use a questo (8).

PROBLEMA 10 ( rapaz ! o que que eu fao aqui ?)


6 amigos encontraram numa expedio um tesouro em que havia N cristais
iguais . Durante a noite um deles vai at o tesouro e faz a diviso dos N cristais por 6.
Feita a diviso, ele percebe que sobra um cristal. Ele pega uma das partes que lhe cabe e
joga fora o cristal que sobrou. Cada um dos outros quatro colegas repete o procedimento,
ou seja, cada um divide a quantidade que est no momento por 6 e sempre sobrando um
cristal, pega a sua parte e joga fora o cristal que sobrou. No dia seguinte, o ltimo repete
o mesmo procedimento e ocorrendo sempre o mesmo fato dos anteriores. Determine o
menor valor possvel de N para que este processo seja possvel .

um problema cujo enunciado semelhante aparece no livro Selecionados de


Matemtica [10], no qual todos os exerccios tm como origem as Olimpadas Americanas.
Sugesto: Faa todas as divises pedidas no problema e perceba algum detalhe de
somar as mesmas quantidades em ambos os membros das igualdades que aparecem.

PROBLEMA 11 ( ainda no dei esta matria !! )


Resolva nos reais a equao : x 4

4x 1 .

Uma questo proposta no livro Problemas Selecionados de Matemtica [15 ] e cujo


objetivo levar o leitor a pensar numa fatorao que simplifique a dificuldade em trabalhar
com uma equao do 4 grau.
12

Sugesto : tente desenvolver algo do tipo a2 = b2 .

PROBLEMA 12 ( estou s olhando pr ela !)


Determine x+y onde x e y so nmeros reais tais que:
4 x 2 9 y 2 4 x 12 y 5 0 .

Questo proposta para utilizar o quadrado de uma soma e a Prop 2.

PROBLEMA 13 ( preciso resolver mais problemas de polinmios !!)


Sabendo que (x-a)(x-10) = (x+b)(x+c)-1 , para todo x real , onde a , b e c so nmeros
inteiros . Encontre a soma dos valores de a .
Do livro The USSR Olympiad Problem Book [16]

PROBLEMA 14 ( est me dando raiva !!)


Determine a diferena entre a maior e a menor raiz real da equao

x2

9 x2
( x 3) 2

27 .

Do livro Problemas Selecionados de Matemtica [15].


Sugesto : quadrado de uma diferena .

PROBLEMA 15 ( por que que no meu livro no tem isto ? e agora ?)


Resolva nos reais : x

2x

15 , onde x representa o maior inteiro menor do que x

ou igual a x .

Esta questo foi proposta com o intuito de mostrar de que forma devemos trabalhar
com variveis em x .

13

PROBLEMA 16 ( deste tipo eu no sabia!)


A igualdade na varivel x : 2mx-x +5 = 3px-2m+p admite as razes
. Determine o valor de m 2

p2 .

Um fato que muita das vezes esquecemos de chamar a ateno dos alunos : uma
equao que foi escrita na varivel x do 1 grau admitindo duas razes, os coeficientes
....

PROBLEMA 17 ( a voc est querendo me derrubar!!)


Determine o inteiro mais prximo da raiz da equao (2 x 3)(4 x 3)( x 1)(4 x 1) 9.

Do livro Problemas Selecionados de Matemtica [15].


Sugesto : tente montar uma equao do 2 grau .

PROBLEMA 18 ( s sabia fazer por produtos notveis!)


Sejam x1 e x2 as razes da equao ax 2 bx c 0 e seja Sn

x1n

x2 n . Prove que aSn

+ bSn-1 +cSn-2 = 0 .Utilize esta igualdade para calcular S2 , S-2 , S3 , S-3 , S-1 .
Esta questo encontra-se no livro descrito no captulo 3 tem (3.6) e denominada de
Frmula de Newton para a equao do 2 grau .
Nos comentrios da soluo feita uma meno recorrncias .

PROBLEMA 19 ( Eu no sei esta tal de Diofantina ! )


Quantas solues inteiras e positivas possui a equao 2x + 3y = 763 ?
Apesar de ser uma equao diofantina, propomos que seja resolvida para o aluno
sem esta teoria.
Sugesto : tire x como funo de y

14

PROBLEMA 20 ( rapaz, eu pensei substituir valores !)


Prove que

4n 2 no inteiro para todo natural n .

Sugesto : Pense no que ocorre com um quadrado perfeito quando o dividimos por 4.

PROBLEMA 21 ( por geometria ? deve ter um traado mgico!)


Seja ABC um tringulo issceles de base BC . Seja P um ponto sobre o lado AC tal que BP
passe pelo circuncentro do tringulo . Sabendo BC=AP , determine os ngulos do tringulo .

Proposta na Magazine Function [ 6] com um bom grau de dificuldade .


Sugesto da revista : use trigonometria (na soluo usamos apenas argumentos
geomtricos).Tente, vale a pena pensar !

PROBLEMA 22 ( muita imaginao!)


Uma escada possui 10 degraus . Uma pessoa sobe at o ltimo degrau na seguinte condio
:subindo um ou dois degraus . De quantas maneiras esta pessoa pode chegar ao ltimo
degrau ?

Interessante pensar nesta questo sem utilizar argumentos combinatrios.


Sugesto : faa para uma quantidade menor de degraus e verifique

o que est

ocorrendo...

PROBLEMA 23 ( instigante hein ! )


O nmero 3 pode ser expresso como a soma de um ou mais naturais de 4 maneiras, contando
com as ordens :
3, 1 + 2, 2 +1, 1+1+1.
De quantas maneiras podemos expressar o natural n ?
Do livro Mathematical Morsels [ 7 ] .
Sugesto : Faa uma fila de n uns e pense nos espaos existentes. Tente
argumentar sem combinatria, isto ser interessante para o aluno do 9 ano do fundamental.

15

PROBLEMA 24 ( at d pra pensar em alguma coisa!)


Seja n um inteiro positivo dado . Se x e y so inteiros positivos tais que xy = 137x + 137y,
calcule o menor e o maior valor possvel de x .
Modificada do livro Do livro Problemas Selecionados de Matemtica [15].

PROBLEMA 25 ( deve ser desigualdade triangular, eu acho!)


Em um quadriltero ABCD convexo , mostre que MN

AB CD
; onde M e N so os
2

pontos mdios BC e AD , respectivamente .

Questo proposta e utilizada como um teorema para resolver uma questo da 13


OBM. Este teorema se encontra no livro do captulo 3 tem ( 3.3).

PROBLEMA 26 ( vou sair no brao!)


Sejam ABCD um quadrado , E mdio de CD e M interior ao quadrado . Sabendo que

MAB

MBC

EMB

x , determine x .

De uma Olimpada Americana.

Sugesto : use a propriedade de que a mediana relativa hipotenusa no tringulo


retngulo, a metade da hipotenusa .

PROBLEMA 27 ( esta de alguma forma sai!)


Considere a equao 2007 x2 4012 x 2005 0 e sejam a e b suas razes . Determine
valor de a-b em mdulo.

Questo que j foi alvo do concurso de acesso ao Colgio Naval .


Sugesto : observe os coeficientes; o objetivo no utilizar a frmula resolvente da
equao.

16

PROBLEMA 28 ( nossa! O que isto meu camarada ?)


Seja S o conjunto soluo , no campo dos reais , da equao

x 3 x4 2x2 1
x2 1

x2 1
x

2
3

Determine o nmero de elementos de S.

Aparentemente uma equao difcil. Idealizada para utilizarmos o problema 8.

PROBLEMA 29 ( esta acho que tambm sai !)


Se ( x 2

x 4) 2 8 x( x 2

x 4) 15 x 2

0 ,determine o nmero de valores reais de x desta

equao .
Do livro Problemas Selecionados de Matemtica [15].

PROBLEMA 30 ( pr que esta complicao!)


Se a, b,c formam um tringulo, prove que ,para todo n=2,3,4,..., n a , n b , n c tambm formam
um tringulo.
Do livro Mathematical Morsels ,pag 52 , On the Lengths of sides of a triangle, [
7].

PROBLEMA 31 ( sei l o que eu vou fazer!)


Para n natural maior do que 1, prove que

1
n

n 1 n 2

...

1
n2

1.

Do livro Mathematical Morsels ,pag 155 , an inequality of reciprocals, [ 7].

17

PROBLEMA 32 ( isto t me cheirando mdias!)


Sejam b1,b2, ...,bn qualquer permutao dos nmeros reais positivos a1,a2, ..., an. Prove que

a1
b1

a2
b2

an
bn

...

n.

Do livro Mathematical Morsels ,pag 83 , fractions by permutation, [ 7].

PROBLEMA 33 ( esses caras ficam inventando!)


Quatro pontos P,Q,R,S pertencem a um crculo de tal forma que o ngulo PSR reto. Sejam
H e K as projees de Q nos segmentos PR e OS, respectivamente. Prove que a reta HK
divide o segmento QS ao meio.
Da revista Eureka! 35 problema proposto nmero 153.

PROBLEMA 34 ( por onde eu comeo ?)


Sejam a,b,c nmeros reais positivos tais que a+b+c=1. Prove que

a bc
a bc

b ac
b ac

c ab
c ab

3
.
2

Da revista Eureka! 35 problema proposto nmero 154.

PROBLEMA 35 ( acho que vou usar a calculadora!)


7

Sem o uso de uma calculadora , qual dos nmeros

3 e 7

o maior ?

Uma questo que pode ser proposta para os alunos que estudam logaritmos, mas que
sugerimos que utilize o fato de que 48 < 49 e somente propriedades de potenciao.

PROBLEMA 36 ( essas desigualdades me deixa pasmo!)


Se a,b,c,d so nmeros reais positivos tais que a soma seja igual a 1, prove que

4a 1

4b 1
4c 1 4 d 1
18

Do livro More Mathematical Morsels[8]. (Problems from the Leningrad High


School Olympiad,1980 ).

PROBLEMA 37 ( vou traar paralelas ao lados do quadrado a partir de P!)


P um ponto interior ao quadrado ABCD tal que PA=1, PB=2 e PC=3. Determine

APB .

Figura 1: (Ponto interno no quadrado)

Do livro More Mathematical Morsels[8]. ( Problem M796 from Kvant(p.269)

PROBLEMA 38 ( esta no d pra mim!)


Prove que a raiz positiva de
x( x 1)( x 2)...( x 2013) 1

menor do que

1
, onde 2013! = 1.2.3.....2013.
2013!

Do livro More Mathematical Morsels[8]. ( Leningrad High School Olympiad,1981)

PROBLEMA 39 ( hum! 121 = 11x11 !)


Seja n um inteiro. Existe algum n tal que n2
19

2n 12 divide 121 ?

Do livro The Canadian Mathematical Olympiad-problem 6-1971)

PROBLEMA 40 ( rapaz , voc tem imaginao!)


Qual o maior inteiro positivo n tal que n +10 divide n3 100 ? .
Do livro More Mathematical Morsels[5]. (Problem from the american Invitational
Mathematics Exam, 1986 )

PROBLEMA 41 ( a primeira eu fao no brao mesmo, mas a segunda , no sei no!)


Uma soluo para diviso de polinmios que facilita em algumas situaes para ser
abordada em sala de aula que causa surpresa :
a) Determine o resto da diviso de P(x) = x3 5 x 1 por D(x) = x 2 3x 1
b) Determine o resto da diviso de P(x) = x2013 5x247 6 x 1 por D(x) = x8 1 .

PROBLEMA 42 ( saindo no brao,acho que d!)


Uma desigualdade instigante : Sejam a,b,c,d nmeros reais . prove que

1 ab

1 cd

ac

bd

1.

Do livro Leningrad Mathematical Olympiads ( The Fifty-Third Olympiad-1987)

PROBLEMA 43 ( caramba!)
Qual o valor mximo possvel da rea de um quadriltero que tem os lados iguais a 1,4,7 e
8?
Do livro Leningrad Mathematical Olympiads ( The Fifty-fifth Olympiad-1987).

PROBLEMA 44 ( isto me assusta!)


Existe algum natural n tal que n n

( n 1) n seja divisvel por 4027 ?

Modificado do livro Leningrad Mathematical Olympiads ( The Fifty-Third


Olympiad-1987).
20

PROBLEMA 45 ( eu vou desenvolver tudo e....!)


Sejam a,b,c,d nmeros reais positivos. Prove que

1
a

1
b

4 16
c d

64
.
a b c d

Do livro Leningrad Mathematical Olympiads (The Fifty-fourth Olympiad-1988).

PROBLEMA 46 ( isto loucura , vou particularizar!)


Escreva 2013 como soma de ( um nmero arbitrrio de) parcelas de modo que o produto das
parcelas seja o maior possvel.
Adaptado da revista Eureka! nmero 1-1998.

PROBLEMA 47 ( o meu camarada! Por que voc faz isto comigo!)


Represente o nmero 989.1001.1007+320 como um produto de primos.
Do livro Leningrad Mathematical Olympiads ( The Fifty-Third Olympiad-1987).

PROBLEMA 48 ( vou sair no brao tambm e vejo o que d no final!)


Um quadriltero tem cada um dos seus vrtices sobre os lados de um quadrado de lado
1.Tomando a,b,c, d como sendo os lados deste quadriltero , prove que

2 a 2 b2 c 2 d 2

4 .

Do livro The Canadian Mathematical Olympiad-problem 5-1970).

21

PROBLEMA 49 ( sem dizer a quantidade de dgitos !)


Prove que no existe um inteiro tal que a retirada do primeiro dgito produz um resultado
que

1
do inteiro original.
35

.
Do livro The Canadian Mathematical Olympiad-problem 4b-1971)

PROBLEMA 50 ( este o mesmo que ganhar na MegaSena!)


Um problema difcil e um resultado interessante : determine o valor de
1 2 1 3 1 4 1 5 ... .

Do livro Problemas selecionados de Matemtica [ 10] .

22

CAPTULO 2 : SOLUES DOS PROBLEMAS


Neste captulo solucionaremos os problemas utilizando apenas o conhecimento de teorias
curriculares do ensino fundamental. Faremos as construes de idias e raciocnios, onde
neles contm desenvolvimentos de contedos programticos na disciplina de Matemtica
Bsica com um formalismo diferente dos livros textos utilizados nas escolas pblicas.

Soluo do Problema 1
possvel dividir uma quadrado em 1993 outros quadradinhos ?esses quadradinhos
no so necessariamente congruentes.

Uma das solues dada observar que dado um quadrado podemos dividi-lo em
quatro quadradinhos menores (Fig. 2) e depois um dos quadradinhos dividi-lo novamente em
quatro outros quadradinhos (Fig. 3) e repetir o procedimento.

Figuras 2 e :3 ( diviso do quadrado em 4 e 7 quadradinhos )

Observe que de um quadrado obtivemos a sequncia de quantidade de


quadrados : 1 ,4 ,7,.... Temos s que verificar se 1993 est nesta seqncia e caso esteja, o
procedimento da diviso ser o descrito acima. Podemos verificar que realmente 1993 est
nesta seqncia. Tente !
Logo a resposta Sim .

23

COMENTRIOS :
(I) A construo que est descrita acima, provavelmente, far o aluno a questionar se h outra
uma construo. O leitor fica convidado a pensar se h uma outra montagem que nos leve
soluo.
(II) Dentro do exposto podemos observar que rascunhar ideias e situaes simplificadas pelo
que foi pedido torna-se importante para uma melhor visualizao.
(III) Aps a montagem da figura, questionar como o aluno conseguiria mostrar que 1993 est
na seqncia sem utilizar a teoria de P.A, interessante. Para o aluno que tenha o
conhecimento de PA, utilizar a expresso do termo geral da PA um fator motivador da
aplicao desta expresso. Neste momento levar o aluno a verificar que na montagem da
figura, a cada quadradinho dividido acrescentamos trs outros quadradinhos na quantidade
anterior e que a razo da P.A .
(IV) Olhando para o enunciado, podemos imaginar a possibilidade de dividir o quadrado em
outra quantidade diferente de 1993. Pense nisto!
(V) Note que esta questo pode ser um bom exemplo de aplicao da expresso do termo
geral da PA , ou seja ,
an

a1 (n 1).r

em que
1993=1+(n-1).3
donde teremos
n=665.

Como n um inteiro, chegamos concluso que 1993 est na sequncia encontrada.

Soluo do Problema 2
possvel dividir uma quadrado em n outros quadradinhos ? esses quadradinhos
no so necessariamente congruentes.

O problema surgiu diante de uma discusso entre professores e cuja soluo abaixo
fruto de vrias ideias.
Observe as divises a seguir :
a) Dividindo o quadrado em 4 e 7 quadradinhos: vide Figs. 2 e 3.
24

Observe que a cada quadradinho so acrescidos 3 quadradinhos e temos a sequncia;


4,7,10,... , que a soluo do problema anterior.
b) Dividindo o quadrado em 6 e 9 quadradinhos

Figuras 4 e 5 ( diviso do quadrado em 6 e 9 quadradinhos)

E utilizando o procedimento anterior teremos a seqncia : 6,9, 12,...

c) Dividindo o quadrado em 8 e 11 quadradinhos

Figuras 6 e 7 ( diviso do quadrado em 8 e 11 quadradinhos)

Observe que teremos a seqncia : 8,11,14,...


Vamos agora escrever separadamente as sequncias obtidas :

1 seqncia : 4,7, 10, 13, 16, ... ( do problema 1)


2 seqncia: 6,9,12,15,...
25

3 sequncia; 8,11,14,17,...
Unindo as trs sequncias obtidas obtemos 4,6,7,8,9,10,11,... que a seqncia dos
nmeros naturais a partir do quatro , com exceo do 5 .

CONCLUSO :
Pela sequncia final obtida verificamos a possibilidade em dividir um quadrado em
qualquer quantidade de outros quadradinhos, no constando a diviso em 5 outros
quadradinhos.

COMENTRIOS :
(I) Utilizando os comentrios do problema 1 podemos entender o porqu das construes .
(II) Fazer com que o aluno pense o motivo pelo qual um quadrado no possa ser dividido em
cinco outros quadradinhos realmente um grande desafio. Deixamos ao leitor tal desafio, o
qual no temos uma resposta afirmativa ou negativa contundente, entretanto, acreditamos que
seja impossvel obter tal diviso.

Soluo do Problema 3
Prove que a soma
S

1 1
1 2

1
3

1
1
...
4
n

n
i

1
1 i

no um valor inteiro .

Observe que na seqncia 1,2,3,4,...,n ; existe um nico inteiro positivo k com


2k

2k 1 . Caso tivssemos na decomposio dos nmeros 2,3,4,...,n em fatores primos

algum nmero com o expoente de 2 superior a k , no poderia ocorrer 2k

2k 1 .

Tomando o mmc dos nmeros 1,2,3,4,...,n , encontraremos uma frao cujo


numerador um nmero mpar (tente mostrar) e o denominador um nmero da forma 2k .I
que par; o que torna o resultado um racional no inteiro.

26

COMENTRIOS:
(I) Observe que o desenvolvimento pode no ser visvel de imediato para um aluno
inexperiente em Olimpadas, portanto interessante pedir ao aluno que faa algumas contas
para n pequeno e enfatizar o termo 2 k e, com isto perceber o que est ocorrendo quando se
coloca sob um mesmo denominador. Verificar que o numerador final um mpar pode no ser
to imediato.
(II) A Prop 1 gera uma certa surpresa at para alunos acostumados em enfrentar determinados
concursos militares e, mais surpresos ainda ficam os que esto se preparando para as
Universidades em geral.
(III) A prova da Prop 1 obtida pela decomposio em fatores primos , pois teremos
n

2k1 3k2 5k3 .... que obviamente um nmero da forma 2k .I , onde I um mpar.

(IV) Um fato que deve ser comentado que a Prop 1 no o determinante na soluo e sim
um endosso curioso que podemos retirar do problema.

Soluo do Problema 4
Considere um paraleleppedo retngulo de lados variveis e reais tais que o permetro
constante igual a P . Determine o volume mximo deste paraleleppedo e suas dimenses .

Fatorao :
a3 b3 c3 3abc

(a b c)(a 2 b 2 c 2 ab ac bc)

com a,b,c nmeros reais.


Podemos escrever a fatorao acima da seguinte forma:
a3 b3 c3 3abc

1
(a b c)((a b) 2 (b c) 2 (a c) 2 ) .
2

Vamos agora considerar a,b,c nmeros reais positivos .


Pela Prop 2 temos que
a b

b c

a c

0,

ou seja,
a 3 b3 c 3

3abc

Observe que a igualdade ocorre quando tivermos a=b=c e tomando a3=x, b3=y e c3=z ,
podemos escrever

27

y z
3

xyz

e concluir a seguinte propriedade :

PROPRIEDADE 3: A mdia aritmtica de nmeros reais positivos maior do que ou igual


mdia geomtrica e, apesar de aparecer somente para trs nmeros reais positivos , provase que a desigualdade vlida para uma quantidade qualquer de nmeros reais positivos
(vide [3]).

Sejam x , y e z as dimenses do paraleleppedo retngulo . Pelo enunciado temos :


4( x

y z)

P e o volume

xyz .Usando Prop. 3 teremos

P
12

e consequentemente
P3
.
123

Observe que o volume mximo


P3
12 3

e cada dimenso nessas condies dada por


x

P
,
12

onde evidentemente temos um cubo.

COMENTRIOS:
(I) Inicialmente importante provar Prop 2 .
(II) Importante tambm provar a fatorao indicada, fatorao esta que em geral no aparece
em livros textos do ensino fundamental ou mdio .
(III) Talvez um aluno de 9o ano ou at mesmo do ensino mdio, no tenha a facilidade de
avaliar que a Prop 2 e a fatorao sejam o caminho para a soluo dada acima. Nos livros ou
artigos de desigualdades indicados no captulo 3 h uma quantidade expressiva de problemas
que utilizam esta propriedade.
(IV) bem provvel que o docente que no tenha a vivncia em preparo para alunos de
Olimpadas de Matemtica, pense logo em usar o clculo diferencial integral. Note que
28

mesmo assim temos um problema de clculo a mais de uma varivel, fato este que nem
sempre lembrado pelo professor que no atua em aulas de clculo.

Soluo do Problema 5
Suponha que tenhamos
Determine o valor da expresso

a,b,c nmeros reais no nulos , tais que


a2
bc

b2
ac

a b c 0.

c2
.
ab

Uma questo proposta em funo do problema 4.


Observe que de acordo com a fatorao do problema 4:
a3 b3 c3 3abc

(a b c)(a 2 b 2 c 2 ab ac bc)

e j que
a+b+c = 0
teremos
a 3 b3 c 3

3abc

e dividindo a expresso por abc ( abc 0) :

a3
abc

a3
abc

a2
bc

b2
ac

a3
abc

ou

c2
ab

3.

COMENTRIOS:
(I) Observe que os passos utilizados na soluo se tornam eficazes com o conhecimento da
fatorao.
(II) interessante tambm notar que um aluno poderia desenvolver as somas das fraes
como a seguir :

29

a2
bc

b2
ac

c 2 a 3 b3 c 3
=
ab
abc

e que atravs da fatorao


a3 b3 c3 3abc

(a b c)(a 2 b 2 c 2 2ab 2ac 2bc)

e j que a b c 0 encontraremos o valor 3 como resposta.


(III) tambm muito importante chamar a ateno do aluno com relao ao fato do enunciado
ter colocado a,b,c reais no nulos , pois esses valores surgem nos denominadores .

Soluo do Problema 6
Qual a quantidade mnima k de pessoas presentes numa reunio, para que
possamos garantir que n pessoas nasceram no mesmo dia da semana ?
Uma questo que tem como objetivo chamar a ateno de um princpio presente em
vrias situaes.
Neste problema interessante comear com pequenos valores para n e verificar
alguma recorrncia:
a) n=2
segunda

Tera

quarta

quinta

sexta

sbado

domingo

X
Tabela 1 (quantidade mnima para n = 2)

Como temos 7 dias na semana e montando o quadro acima , temos k=8


b) n=3
segunda

tera

quarta

quinta

sexta

sbado

domingo

X
Tabela 2 (quantidade mnima para n = 3)

Usando o mesmo argumento do caso (a), observe que temos k = 15.


Observando o que ocorre nos dois casos particulares temos
k = 7(n-1)+1 .
30

COMENTRIOS :
(I) interessante deixar que o aluno pense nas situaes (a) e (b), e logo depois conclua a
expresso acima e caso ele tenha dificuldade em concluir, ajud-lo na construo da
expresso.
(II) Podemos neste momento comentar que na verdade se trata de um problema envolvendo o
Princpio das Casas dos Pombos que denominaremos aqui de PCP .Enunciar o princpio
talvez seja instigante para o aluno.
(III) O grande desafio no PCP como utiliz-lo em problemas que fazem partes dos livros do
Captulo 3.

Soluo do Problema 7
Seja ABC um tringulo tal que AB=AC e que
Determine

BAC

200 como na figura.

CPQ .

Figura 8 ( um antigo problema que nos deixa perplexos)

Este problema possui um detalhe interessante em relao Lei angular de Tales; no


se consegue o valor do ngulo trabalhando apenas com esta lei nos tringulos que aparecem
no interior de ABC.
31

Traando CR como indicado na figura 10 e utilizando a Lei angular de Tales(a soma


dos ngulos internos de qualquer tringulo igual a 180), teremos as seguintes concluses :
BC=CQ( tringuloBCQ issceles)

(1)

CR=BC( tringulo BCR issceles)

(2)

CR= PR ( tringulo CRP issceles)

(3)

De (1) e (2) podemos concluir que


CR= CQ

(4)

e consequentemente o tringulo CQR issceles.


Como no tringulo CQR o ngulo interno de vrtice C de 60 , conclumos que o
tringulo CQR na verdade equiltero( todo tringulo issceles que possui o ngulo interno
de lados congruentes igual a 60 equiltero), donde podemos concluir que
CR=CQ=QR

(5).

De (3) e (5) conlumos que QR=PR, logo o tringulo QRP issceles .Como neste
tringulo o ngulo interno de vrtice R igual a 40 e que os ngulos internos de vrtices Q
e P so iguais , teremos :
40+40+x+40+x =180 e donde x = 30.

Figura 9 ( que genialidade!)

32

COMENTRIOS :
(I) Podemos ver que dificilmente um aluno que no conhea a soluo pensaria da forma que
foi desenvolvida.
(II) interessante que no momento que se traa o segmento CR como na figura 2 , deixar o
aluno tirar as concluses e acompanh-lo at chegar o valor do ngulo pedido.
(III) um problema muito difundido para os participantes de Olimpadas e vrias solues j
surgiram para a questo, inclusive trigonomtrica ( para alunos com alguns conhecimentos
nesta disciplina), vide Apndice E.
(IV) Os problemas de tringulos que envolvem segmentos e clculo de ngulos inteiros como
o valor encontrado de x so denominados de ngulos adventcios (vide Apndice D).

Soluo do Problema 8
1
x

Prove que x

1
x

2 ou x

0 e real

2,x

Utilizaremos esse problema como uma propriedade, pois a utilizaremos em outros


deste captulo.

PROPRIEDADE 4 :
x

1
x

2 ou x

1
x

2,x

Prova :
Seja m= x

1
x2 mx 1 0
x

Como x um real, devemos ter o 0 , ou seja m2 4 0 .Resolvendo a inequao em


m temos m -2 ou m 2 , que traduz a propriedade (4) .

COMENTRIOS :
(I) Esta propriedade pode ser escrita da seguinte forma :
33

PROPRIEDADE 4': Um nmero real ( no nulo) somado com o seu inverso , maior do que
2 ou igual a 2 ; ou menor do que -2 ou igual a -2 .

(II) Interessante mostrar que as igualdades ocorrem para x =1 quando x positivo e x = -1


quando x negativo .
(III) Observe que para x positivo , podemos utilizar a desigualdade das mdias para os dois
reais x e

1
(Prop. 3).
x

1
x

x.

1
1
x
x
x

2 .

(IV) Interessante tambm mostrar que utilizando produtos notveis e a Prop. 2 teremos, nas
condies de (III) que
2

1
x

0 x

1
x

2 .

Soluo do Problema 9
13

Resolva nos reais a equao :

13

x12
x

13

Observe que a primeira parcela da soma o inverso da segunda e, como

13

7 uma

valor entre 1 e 2, podemos utilizar a Prop 4 para garantir que no h soluo nos reais .

COMENTRIOS :
(I) A observao de que temos a soma com o seu inverso pode no ser bvio , portanto
chamar a ateno com relao a este fato talvez torna-se necessrio.
(II) Aps ter a viso do inverso interessante mostrar a seguinte soluo ;
Tomando y

13

x , teremos

34

1
y

13

e consequentemente
y2

13

7y 1 0

no possui razes reais.


(III) Interessante tambm justificar o fato de que

13

7 um valor entre 1 e 2.

Observe que
1<7<213
e consequentemente
1< 13 7 <2 .

Soluo do Problema 10
6 amigos encontraram numa expedio um tesouro em que havia N cristais
iguais . Durante a noite um deles vai at o tesouro e faz a diviso dos N cristais por 6.
Feita a diviso, ele percebe que sobra um cristal. Ele pega uma das partes que lhe cabe e
joga fora o que sobrou. O procedimento repetido por cada um dos outros quatro colegas,
ou seja, cada um divide a quantidade que est no momento por 6 e sempre sobrando um
cristal, pega a sua parte e joga fora o cristal que sobrou . No dia seguinte, o ltimo repete
o mesmo procedimento e ocorrendo sempre o mesmo fato dos anteriores .Determine o
menor valor possvel de N para que este processo seja possvel .

Esse problema demonstra para o leitor uma originalidade extraordinria. Aps


encontrarmos o valor de N verificamos que no um problema que possa ocorrer na vida real
como descreveremos a seguir.

35

Pelo enunciado :

6q1 1 e quando o segundo chega para fazer a diviso

encontra 5q1 moedas, donde :


5q1

6q2 1

5q2

6q3 1

5q3

6q4 1

5q4

6q5 1

5q5

6q6 1
Somando 5 em ambos os lados de cada igualdade teremos :

N 5 6(q1 1)
5(q1 1) 6(q2 1)
5(q2 1) 6( q3 1)
5(q3 1) 6( q4 1)
5(q4 1) 6( q5 1)
5(q5 1) 6(q6 1)
Multiplicando todas as seis igualdades teremos :

55 ( N

5)

66 ( q6 1) .

Observe que temos somente inteiros em ambos os lados da igualdade e


q6 1

55 ( N 5)
66

ou seja,
N

66.t 5

com t sendo um inteiro positivo.

Conclumos que o menor valor de N igual a ( 66 1 ) =46651.

COMENTRIOS :
(I) A tentativa inicial em substituir e relacionar as igualdades geralmente se torna um trabalho
rduo que em geral se desiste no meio do caminho.

36

(II) Ter a viso de adicionar 5 unidades em ambos os lados das igualdades mostra uma
sacada1 extraordinria, ou seja, tal ideia mostra que a experincia em se resolver muitos
de problemas um fator relevante.
(III) Podemos neste problema comentar a definio de nmeros primos entre si ou primos
relativos, pois 5 e 6 so primos entre si, apesar de 6 no ser primo .

Soluo do Problema 11
Resolva nos reais a equao : x 4

4x 1 .

tambm uma questo do livro [6] e de imediato o leitor poder perder muito
tempo em tentar buscar alguma propriedade que leve soluo.
Note que podemos escrever a equao da seguinte forma :
x2 1

2( x 1)2 e consequentemente x 2 1

2( x 1) .

Resolvendo as duas ltimas igualdades, encontraremos apenas uma equao do 2 com


razes reais.Teremos apenas dois valores reais para o conjunto soluo.

COMENTRIOS :
(I) interessante observar que um aluno que nunca resolveu uma equao desse tipo,
certamente ter dificuldade em completar os quadrados.
(II) Induzir o aluno a tentar completar quadrados em ambos os lados da equao, pode fazer
com que ele chegue na soluo acima.
(III) Um aluno que tenha conhecimento de equaes polinomiais poderia tentar conseguir
solues particulares, fato este que fatalmente o levaria a desistir, j que as solues no so
racionais
(IV) Aps o aluno perceber o que ocorreu na soluo, desafiar o aluno a construir outros tipos
de equaes que nos leve a solues semelhantes pode ser um fator bastante agradvel para o
mesmo.
(V) Talvez seja interessante para um aluno que conhea algumas construes grficas indicar
a quantidade de solues encontradas como a interseco dos grficos das funes g(x) = x4 e
f(x) = 4x+1 (veja a seguir).
1

sacada a expresso utilizada pelos alunos do PROFMAT da minha turma para dizer que algum teve uma ideia brilhante com
relao a um problema

37

Figura 10 ( verificando se h soluo)

Soluo do Problema 12
Determine x+y onde x e y so nmeros reais tais que:
4 x 2 9 y 2 4 x 12 y 5 0 .

uma questo para justamente chamar ateno com relao aos quadrados que
aparecem no incio da equao .A dificuldade inicial que temos duas variveis.
Observe que podemos escrever a equao da seguinte forma :
2x 1

3y 2

Utilizando P2 verificamos que devemos ter obrigatoriamente :


2x-1=0 e 3y-2=0 , ou seja
x

1
ou
2
x

38

y
7
.
6

2
3

COMENTRIOS :
(I)

Apesar de usar uma propriedade simples, a igualdade com duas variveis pode parecer

um exerccio difcil para alguns.


(II)

Para alunos que conhecem o estudo de cnicas provavelmente teriam a ideia de

completar os quadrados.
(III) Chamar ateno para o fato de que a soma dos quadrados de dois nmeros reais s zero
quando ambos so nulos.

Soluo do Problema 13
Sabendo que (x-a)(x-10) = (x+b)(x+c)-1 , para todo x real , onde a , b e c so nmeros
inteiros . Encontre a soma dos valores de a .

um problema de uma Olimpada Russa [16] ,que dependendo do caminho a ser


seguido, a soluo poder ser no muito trivial.
Como temos uma identidade , j que a igualdade ocorre para todo x real , podemos em
ambos os lados da igualdade colocar x = -b e consequentemente :
(b+a)(b+10) = -1.
E como a, b e c so inteiros ,devemos ter :
1 hiptese : b+a = 1 e b+10 = -1 , ou seja b= -11 e a =12.
2 hiptese : b+a = -1 e b+10 = 1 , ou seja b= -9 e a = 8.
Soma = 18.

COMENTRIOS :
(I) Observe que se substitussemos x = c encontraramos as mesmas solues. Tente explicar
este argumento.
(II) Interessante pedir ao aluno para pensar na identidade de dois polinmios para encontrar
os valores de a.

39

Soluo do Problema 14
Determine a diferena entre a maior e a menor raiz real da equao

9 x2
( x 3) 2

Esta questo foi de uma Olimpada

27 .

Americana e em geral traduz uma grande

dificuldade para muitos que tentaram solucion-la.


Observe o desenvolvimento a seguir :
3x
x 3

6x2
x 3

x2

9x2
x 3

(6)

Desenvolvendo o lado esquerdo da igualdade acima , teremos :


x2
x 3

e observando que x 2

9 x2
( x 3) 2

6 x2
x 3

9x2
x 3

27 e fazendo y

a equao (7) ficar da seguinte forma

y2

(7)

x2
x 3

6 y 27 ou y 2 6 y 27

0.

Teremos ento y = -9 ou y = 3 e encontrando ento os valores reais de x , a diferena


pedida ser igual a 3 5 .

COMENTRIOS :
(I) Observe que a soluo apresentada , apesar de utilizar produtos notveis , se faz de uma
forma sofisticadssima e portanto um aprendizado e tanto.
(II) Orientar o aluno a pensar no quadrado de uma diferena e tentar induzi-lo a chegar na
soluo acima , pode ser gratificante para ele.

Soluo do Problema 15
Resolva nos reais : x

2x

15 , onde x representa o maior inteiro menor do que x

ou igual a x .

40

Podemos perceber que

x =5 soluo da equao e como a varivel x no

necessariamente inteira , por verificao temos que o intervalo [5, 11/2[ a soluo da
equao.
claro que a ideia acima seria vivel para uma prova de mltipla escolha na qual se
ganha tempo. Uma forma geral de solucionar o problema descrita a seguir.
Sejam x

k e y tais que x = k + y , onde 0 y < 1 e 2x = 2k + 2y .

Como 0 y < 1 teremos 0 2y < 2 .


Temos ento duas hipteses :
1 hiptese : 0 2y < 1 ou 0 y <
2k + 1, ou seja, x
A equao

1
, logo k < x = k + y < k + 1 e 2k < 2x =2 k + 2y <
2

k e
2x

2x

k .

15 ser escrita como 3k =15 ou k = 5.

Temos neste caso ento a soluo x = 5 + y com 0 y <


2 hiptese : 1 2y < 2 ou

1
y < 1 , logo logo k + 1/2 < x = k + y < k + 1 e
2

2k + 1 < 2x =2 k + 2y < 2k + 2, ou seja, x


A equao

1
.
2

2x

k e 2x

2k 1 .

15 ser escrita como 3k + 1 =15 ou k =

14
, que no
3

inteiro.

Descartamos ento esta segunda hiptese.


A resposta do problema o intervalo

5,

11
.
2

COMENTRIOS :
(I) Pedir ao aluno que tente experimentar valores pode ajudar para que ele tenha alguma
intuio em presseguir a soluo descrita.
(II) Observar que a dificuldade pode se estender se tivermos um termo do 2 grau. Tente
exemplificar.
(III) Tente resolver a equao a seguir utilizando a mesma ideia : x + 2 x

41

3x

Soluo do Problema 16
3

A igualdade na varivel x : 2mx-x +5 = 3px-2m+p admite as razes


Determine o valor de m 2

2, 3 3

3 .

p2 .

A equao originalmente dada na varivel x do 1 grau , ou seja teremos apenas uma


raiz. Como o enunciado admite duas razes e escrevendo a equao original na forma
(2m-3p-1)x = p-5-2m,

Devemos ter obrigatoriamente :

2m 3 p 1 0
e

e resolvendo o sistema encontramos m = - 4 e p = - 3.

p 2m 5 0
Logo m2

p2

25 .

COMENTRIO:
Nos livros textos bsicos em geral no so apresentadas equaes com este formato e o aluno
fica perplexo com este tipo de equao.

Soluo do Problema 17
Determine

inteiro

mais

prximo

da

maior

raiz

da

equao

(2 x 3)(4 x 3)( x 1)(4 x 1) 9 .

Multiplicando o 1 binmio com o terceiro encontraremos : 2 x 2


multiplicando o 2 binmio com o quarto : 16 x2 8 x 3 . Fazendo y
original ficar escrita da seguinte forma : y 3 2 y 3
que nos fornece y

0 ou y

2x2

x 3

x , a equao

9 em que teremos 2 y 2 9 y 0 , o

9
. Voltando para a varivel x e resolvendo as equaes do 2
2

grau teremos como resposta do problema o valor 2.

42

COMENTRIOS :
(I) pedir o aluno a desenvolver o produto interessante para perceber a dificuldade final.
(II) Tentar uma outra multiplicao diferente da apresentada pode surgir uma ideia diferente,
tente ....

Soluo do Problema 18
x1n

Sejam x1 e x2 as razes da equao ax 2 bx c 0 e seja Sn

x2 n .Prove que aSn

+ bSn-1 +cSn-2 = 0. Utilize esta igualdade para calcular S2 , S-2 , S3 , S-3 , S-1 .
Como x1 e x2 so razes da equao ax 2 bx c 0 teremos
ax12

bx1 c

(8)

ax2 2

bx2

0.

(9)

Multiplicando (8) por x1n

e (9) por x2n

ax1n

bx1n

(10)

ax2 n

bx2n

(11)

encontraremos

a ( x1n

Somando (10) e (11) :

x2 n ) b( x1n

x2 n 1 ) c

aSn + bSn-1 +cSn-2 = 0.

0 ou

(12)

Em (12) fazendo n =2 , teremos: aS2 + bS1 +cS0 = 0 e sabendo que S1= b e S0 = 2.


a
Encontramos S2 =

b2

2ac
a

= S12 2 P , tomando P =

c
.
a

Em (12) fazendo n=3, teremos aS3 + bS2 +cS1 = 0

e conhecendo S1 e S2 ,

determinamos o valor de S3 = S1( S12 3P ). Tente agora realizar as operaes para encontrar
as outras somas pedidas.
43

COMENTRIO :
Em geral esas somas so desenvolvidas por fatorao sem a recorrncia apresentada.
interesante que faa com o aluno por produtos notveis ou fatorao.

Soluo do Problema 19
Quantas solues inteiras e positivas possui a equao 2x + 3y = 763 ?

A equao pode ser escrita como x

763 3 y
e j que x um inteiro , devemos ter
2

(763-3y) par , ou seja , 3y deve ser mpar , donde y tem que ser um mpar.
763
e como y mpar, temos que y {1, 3, 5,
3
...,253}. Conclumos ento que h 127 valores possveis para y e consequentemente 127 pares

Observe que 763-3y> 0 ou y <

(x,y) como solues da equao.

COMENTRIO :
Dependendo dos coeficientes das variveis , a soluo acima pode ser invivel. Aqui est o
bom momento para parar e estudar com os alunos as equaes diofantinas. Com certeza
ser muito gratificante resolver este e outros problemas com essa teoria.

Soluo do Problema 20
Prove que

4n 2 no inteiro para todo natural n .

PROPRIEDADE 5: um quadrado de um inteiro sempre da form 4k ou 4k+1 .


Prova : Seja x=2t ento x2 = 4t2=4k .
Seja x = 2t + 1 ento x2 = 4t2 + 4t + 1 = 4s + 1.
Utilizando a Prop. 5, conclumos que 4n+2
4n 2 no um inteiro.

44

nunca um quadrado perfeito, da

COMENTRIO :
Repare que voc pode sugerir uma infinidade de radicandos com este formato e que no tenha
raiz exata.

Soluo do Problema 21
Seja ABC um tringulo em que AB=AC . Seja P um ponto sobre o lado AC tal que BP passe
pelo circuncentro O do tringulo . Sabendo BC=AP , determine os ngulos do tringulo .

Figura 11 ( um circuncentro e tanto ! )

Como o tringulo issceles a altura traada do vrtice A passa pelo circuncentro O e


pelo ponto mdio M de BC.
Tracemos AM (figura 12) e seja

BAO , ento

PAO e 2

BOM , j que

AO=BO=OC.

Seja Q sobre OM tal que

OBQ . Observe agora que os tringulos AOP e BOQ

so congruentes ( caso ALA)( figura 13), ou seja BQ = AP. Teremos que o tringulo BQC
ser eqiltero.
Note que o ngulo BQM = 3 e consequentemente teremos 6 = 60 , ou seja = 10.
Logo

BAC 20

ABC

ACB 80 .
45

Figura 12 ( uma soluo geomtrica divina)

COMENTRIO :
A incessante busca foi para uma soluo geomtrica . Tente por trigonometria tambm
muito gratificante ! ( use a Lei dos Senos e transformaes de soma em produto e viceversa)

Soluo do Problema 22
Uma escada possui 10 degraus . Uma pessoa sobe at o ltimo degrau com a seguinte
condio: subindo um ou dois degraus . De quantas maneiras esta pessoa pode chegar ao
ltimo degrau ?

Pelas condies do enunciado, podemos imaginar duas situaes :

Primeira: alcanando o nono degrau(que o penltimo), a pessoa conseguir chegar


ao dcimo, subindo mais um degrau.
Segunda: alcanando o oitavo degrau(que antepenltimo), a pessoa conseguir
chegar ao dcimo, subindo um ou dois degraus.
Agora conclua que :
46

1) O nmero de maneiras de chegar ao dcimo o nmero de maneiras de chegar ao oitavo


somado com o nmero de maneiras de chegar ao nono.
2) O que est descrito em (1) usando o fato apenas do penltimo e antepenltimo ,
independe do nmero de degraus ( claro que devemos ter no mnimo um degrau).
Podemos ento resolver o problema utilizando quantidade de degraus menores e
tentando verificar algum detalhe nos resultados . Veja a seguir ;
Tomando n como sendo o nmero de degraus ,
para n=1, o nmero de maneiras 1.
para n=2, o nmero de maneiras igual a 2.
para n=3, utilizamos a concluso (1), que nos dar 1+2=3
para n=4, utilizamos a concluso (1), que nos dar 3+2=5 .
Observe que temos a seguinte seqncia de valores : 1,2,3,5,8,... , na qual estamos
sempre somando o penltimo com o antepenltimo para conseguir o prximo e que cada um
deles representa a quantidade de maneiras de chegar ao degrau n. Isto no interessante ?
Fazendo as contas at n=10 : 1,2,3,5,8,13,21,34,55,89, obtemos como resposta : 89.

COMENTRIO:
Tambm um excelente momento para estudar com os alunos Recorrncias.....

Soluo do Problema 23
O nmero 3 pode ser expresso como a soma de um ou mais naturais de 4 maneiras,
contando com as ordens :
3, 1 + 2, 2 +1, 1+1+1.
De quantas maneiras podemos expressar o natural n ?
Considere uma fileira de n uns como abaixo:

11111111111...1
Observe que entre os algarismos h n-1 espaos e tomando , por exemplo, trs
desses teremos :

11111111111...1 .
3

47

n 9

Podemos escrever n = 3 + 4 + 2 + n 9 .
Agora ns temos que escolher ou no qualquer um dos espaos internos e colocar
uma barra.
Conclumos ento que

2n-1

formas de escrever o natural n como no

enunciado.

COMENTRIOS :
(I) Tentar argumentar com os alunos sem o uso de combinatria.
(II) Porque no estudar agora com os alunos o Princpio Multiplicativo? Com certeza eles
vo adorar ! .

Soluo do Problema 24
Seja n um inteiro positivo dado . Se x e y so inteiros positivos tais que
xy 137 x 137 y ,

calcule o menor e o maior valor possvel de x .

Resolvendo em y teremos :
y

137 x
com x 137.
x 137

e reescrevendo esta igualdade


y

137 x 137 2 137 2


x 137

137

137 2
.
x 137

Observe que para que y seja um inteiro, (x-137) deve ser divisor de 1372 , e como
137 primo devemos ter :
x-137 = -1 ou x-137 = 1 ou x-137 = -137 ou x-137 = 137
ou x -137 = - 1372 ou x 137 =1372.
Como x um inteiro positivo, o menor valor 136 e o maior valor 1372 + 137.

COMENTRIOS:
(I) refaa o problema com solues inteiras positivas ou negativas, interessante tambm!
(II) pea ao aluno para refazer o problema com n natural no lugar de 137.
48

Soluo do Problema 25
AB CD
; onde M e N so os
2

Em um quadriltero ABCD convexo , mostre que MN


pontos mdios de BC e de AD , respectivamente .

N
M

Figura 13 ( um quadriltero que no se esquece)

Tracemos as diagonais AC e BD. Seja Q o ponto mdio de BD como na figura 2.


Como NQ e MQ so bases mdias no tringulos ABD e BCD , respectivamente , temos que

NQ

AB
com NQ paralelo a AB e MQ
2

CD
com MQ paralelo a CD.
2

Temos agora duas situaes :


Primeira : AB e CD no so paralelos e como conseqncia ( pela existncia de
tringulo) que MN< NQ + MQ ou

MN <

AB CD
.
2

Segunda : AB e CD so paralelos . Teremos M, Q e N alinhados e a igualdade MN =


NQ + MQ ocorrer .

Concluso : MN

AB CD
2

49

Figura 14 ( muito interessante! )

COMENTRIO :
Pea ao aluno para demonstrar para os pontos mdios dos outros dois lados. Isto lhe trar uma
certa segurana neste tipo de problema.

Soluo do Problema 26
Sejam ABCD um quadrado , E mdio de CD e M interior ao quadrado . Sabendo que

MAB

MBC

EMB

x , determine x .

Pelos dados do enunciado conclumos que o tringulo AMB retngulo. Seja N o


ponto mdio de AB. Na Fig 15 trace MN e EN, e sabendo que num tringulo retngulo a
mediana relativa hipotenusa a metade da hipotenusa,tem-se que AN=MN=NB.
Pela Fig. 16 temos que o tringulo EMN tambm retngulo . Como EN = 2.MN , os
ngulos agudos do tringulo EMN so 30 e 60.

50

Figura 15 ( que ponto particular!)

Como pela Fig 16

MNE

2x 90 encontramos 2x 90 60 , ou seja, x 750 .

Figura 16 ( muita observao!)

COMENTRIOS :
(I) demonstre para o aluno a relevncia do tringulo retngulo de ngulos agudos iguais a 30
e 60.
(II) Observamos como uma propriedade simples resolve brilhantemente a questo, no
verdade?

51

Soluo do Problema 27
Considere a equao 2007 x2 4012 x 2005 0 e sejam a e b suas razes . Determine o
valor de a-b.

Notando-se que a soma dos coeficientes da equao zero , podemos utilizar a Prop.
4 e concluir que uma das razes, por exemplo, a=1. Como o produto das razes da equao

Ax 2

Bx C

0 dada por

C
2005
, conclumos que b =
( verifique).
A
2007

COMENTRIOS:
(I) Pea ao aluno para desenvolver pela frmula resolvente e verifique a reao dele.
(II) Com esta questo podemos idealizar infinidade de outras questes neste mesmo estilo.

Soluo do Problema 28
Seja S o conjunto soluo , no campo dos reais , da equao
3

x2 1
x

x 3 x4 2x2 1
x2 1

2
3

O nmero de elementos de S

Observando a igualdade dada que na soma esquerda, um dos termos o inverso do


outro. Note tambm que do lado direito temos um valor

positivo e menor do que 2.

Utilizando a Prop 4 , temos que no soluo h no campo dos reais.


Logo o nmero de elementos de S zero .

COMENTRIO:
Reveja os comentrios da questo 9.
No interessante, como uma propriedade simples resolve o problema ?

52

Soluo do Problema 29
Se ( x 2

x 4) 2 8 x( x 2

x 4) 15 x 2

0 ,determine o nmero de valores reais de x desta

equao .

Seja y

x2

x 4 , a equao ficar escrita como y 2 8 xy 15 x 2

0 , uma equao

do segundo grau em y cujas razes so : 3x e 5x.


Consequentemente teremos as equaes : 3x

x2

x 4 ou 5x

x2

x 4 . Continue

o desenvolvimento .....

COMENTRIO :
Acredito que neste momento, com tudo que foi desenvolvido antes, o aluno j estaria
pensando como na resoluo dada, no ?

Soluo do Problema 30
Se a, b,c formam um tringulo, prove que ,para todo n=2,3,4,..., n a , n b , n c tambm formam
um tringulo.

Se a, b, c formam um tringulo ,ento a + b > c . Consequentemente

ou seja,

c , o que queramos provar.

a+b

c=

COMENTRIO :
Verifique a desigualdade para n=2 e n=3 e isto facilitar a ideia final.

53

Soluo do Problema 31
Para n natural maior do que 1, prove que

1
n

...

n 1 n 2

1
n2

1.

Observe que para n natural e maior do que 1 :


1
n

n 1 n 2

...

1
n2

1
n

...

1
n2

1
n2

1
n2

1
n2

...

1
n2

Ou seja,
1
n

n 1 n 2

1
n

1 2
(n n) 1 .
n2

COMENTRIOS :
(I) tomar alguns valores para n nos trar uma boa viso para prosseguir com a soluo geral
(II) Mostre para o aluno os motivos : n < n2 , n+1 < n2 , ... , e depois faa os inversos pra
finalmente realizar as somas.

Soluo do Problema 32
Sejam b1,b2, ...,bn qualquer permutao dos nmeros reais positivos a1,a2, ..., an. Prove que
a1
b1

a2
b2

an
bn

...

n.

Notamos que utilizando a Prop 3 a resoluo se torna imediata :

a1
b1

a2
b2

...

an
bn

n
pois uma sequncia a outra numa certa ordem.

54

a1 a2
a
. ..... n
b1 b2
bn

1,

NOTA: Faa a partir daqui os seus prprios comentrios e nos envie suas sugestes.

Soluo do Problema 33
Quatro pontos P,Q,R,S pertencem a um crculo de tal forma que o ngulo PSR reto. Sejam
H e K as projees de Q nos segmentos PR e OS, respectivamente. Prove que a reta HK
divide o segmento QS ao meio.

Figura 17 ( uma reta abenoada )

Pelo enunciado, temos que PR um dimetro. Tomando o ponto Q e as as projees H


e K nos segmentos PR e OS , temos que a reta que passa pelos pontos H e K a reta de
Simson e, consequentemente, a interseco de HK com a reta SR se d com a projeo T
de Q sobre a reta SR, ou seja,

QTS

90 . Observe agora que teremos o quadriltero QTSK

sendo um retngulo; donde QS e KT se cortam ao meio.

55

Soluo do Problema 34
Sejam a,b,c nmeros reais positivos tais que a+b+c=1. Prove que

a bc
a bc

b ac
b ac

c ab
c ab

3
.
2

Provar a desigualdade equivalente a provar que

2bc
2ac
2ab
1
1
a bc
b ac
c ab

3
2

que equivalente
ab
c ab

ac
b ac

bc
a bc

3
4

(13)

e j que a+b+c=1 , teremos

c ab (1 a)(1 b)
a bc (1 b)(1 c)

Observe agora que (13) ficar escrita da seguinte forma

ab
(1 a)(1 b)

ac
(1 a)(1 c)

bc
(1 b)(1 c)

3
.
4

(14)

Desenvolvendo (14) e usando o fato de que a+b+c =1 , chegaremos

ab bc ac 9abc ou
1
a

1 1
b c

9 .

(15)

Note que provar a desigualdade do enunciado equivalente a provar (15) , usando a Prop 4
teremos

a b c
3

1
a

E como a+b+c=1 , encontramos


56

3
1
b

1
c

1
a

1 1
b c

Que a desigualdade (15).

Soluo do Problema 35
7

Sem o uso de uma calculadora, qual dos nmeros

3 e 7

o maior ?

Note inicialmente que por 48 49 , temos que

4 3

(16)

Observe que
4 3

(17)

Como

teremos

e usando (17) :

4 3

(18)

Usando (1) em (3) , conclumos que

4 3

.
Logo o maior 7

57

Soluo do Problema 36
Se a,b,c,d so nmeros reais positivos tais que a soma seja igual a 1, prove que
4a 1

4b 1

4c 1 4 d 1

Vamos utilizar novamente a Prop 3 para os nmeros 1 e 4a+1 , com a positivo,


teremos:

1 4a 1
2

1.(4a 1) ou

4a 1 2a 1 .

Observe que a igualdade no ocorrer j que a no nulo . Logo,

4a 1 2a 1 .

Fazendo o mesmo para b, c e d, e adicionando as desigualdades ,teremos :

4a 1

4b 1

4c 1

4d 1 2a 1 2b 1 2c 2d 1

ou
4a 1

4b 1

4c 1

4d 1

2( a b c d ) 4

e como

a b c d 1
4a 1

4b 1

4c 1

4d 1 2.1 4 6 .

Soluo do Problema 37
P um ponto interior ao quadrado ABCD tal que PA=1, PB=2 e PC=3. Determine

58

APB .

Figura 18 ( qual a ideia? )

Considere a construo como na fig 2 , na qual traamos o quadrado ABEF simtrico


de ABCD em relao ao lado AB. BP foi traado de forma que BQ = BP e

QBP 90 e

teremos o lado PQ = 2 2 .

Figura 19 ( s poderia ser de Murray Klamkin )

Observe agora que

ABP

QBE e os tringulos EQB e ABP so congruentes (

caso LAL) e, isto nos garante que EQ = AP = 1. Fato semelhante ocorre para os tringulos
ABQ e BCP, ou seja, AQ = PC = 3. Podemos verificar que
59

AQ

AP

PQ

e como

o Teorema de Pitgoras se e somente se,


consequentemente

conclumos que

QBP 90

APB 135 .

( SOLUO DE MURRAY KLAMKIN)

Soluo do Problema 38
Prove que a raiz positiva de
x( x 1)( x 2)...( x 2013) 1

menor do que

1
, onde 2013! = 1.2.3.....2013.
2013!

Pela equao inicial

1
.
( x 1)( x 2)...( x 2013)

(19)

Como x um valor positivo podemos escrever


( x 1)( x 2)...( x 2013)

Ou
1
( x 1)( x 2)...( x 2013)

1
1.2.3.....2013

(20) em (19) :

1
.
1.2.3.....2013

60

(20)

Soluo do Problema 39
Seja n um inteiro. Existe algum n tal que n2

2n 12 divide 121 ?

Suponha que tenhamos


n2 2n 12 121k .

Ento

n 1

11(11k 1) .

J que 11 primo e divide (n+1)2 conclumos que 11 divide (n+1) .....( tente provar
isto).
Logo 112 divide n 1

11(11k 1) , ou 11 divide (11k-1) . Claramente impossvel.

Resposta : No.

Soluo do Problema 40
Qual o maior inteiro positivo n tal que n +10 divide n3 100 ? .

Escreva a seguinte relao:


n3 100 n3 1000 900 .

A motivao em escrever a igualdade acima desta forma foi devido ao fato de que
(a b) divide ( a 3 b3 ) .

Como (n+10) divide (n3 1000) devemos ter necessariamente que (n+10) divida 900
Logo nmximo = 890 .

Soluo do Problema 41
Uma discusso para ser realizada em sala de aula para alunos que conhecem muitas teorias
no abordadas no ensino Fundamental e mdio :
a) Determine o resto da diviso de P(x) = x3 5 x 1 por D(x) = x 2 3x 1

Para P(x) vamos substituir x 2 por -3x-1 .

61

Observe que P(x) = x3 5 x 1 = x.x 2 5 x 1 . fazendo a substituio encontramos


x.x 2 5 x 1 x(-3x-1) + 5x+1= -3x2 x +5x +1 = -3x2 + 4x + 1.

(21)

Novamente em (21) substituindo x 2 por -3x-1 , teremos


-3x2 + 4x + 1 = -3(-3x-1) + 4x + 1= 13x + 4.

O resto na diviso de P(x) por D(x) ser R(x) = 13x + 4 .


Faa agora a diviso pelo procedimento encontrado em livros bsicos e verifique que o
resultado ser o mesmo.

b) Determine o resto da diviso de P(x) = x2013 5x247 6 x 1 por D(x) = x8 1 .

Vamos utilizar o mesmo procedimento do item (a) :


Como 2013 = 8.251+5 e 247 = 8.30+7 , teremos

x 2013 5 x 247

6 x 1 ( x8 ) 251.x 5 5( x8 )30 .x 7 6 x 1

Em (22) substituindo x8 por (-1), teremos :

( x8 )251.x5 5( x8 )30 .x 7

( 1) 251.x5 5( 1)30 .x 7

6x 1

( 1) 251.x5 5( 1)30 .x 7

6x 1

x5 5 x 7

E o resto na diviso de P(x) por D(x) ser


R( x)

5x7

62

x5 6 x 1 .

6x 1

6x 1

(22)

Soluo do Problema 42
Uma desigualdade instigante : Sejam a,b,c,d nmeros reais . prove que

1 ab

1 cd

ac

bd

1.

Podemos escrever a seguinte igualdade:

1 ab

1 cd

ac

bd

1 2ab (ab)2 1 2cd (cd )2

1 2ab (ab)2 1 2cd (cd )2


ac

bd

1 ab cd

ac

ac bd

bd
2

Pela Prop 2:

1 ab cd

ac bd

1 1.

Soluo do Problema 43
Qual o valor mximo possvel da rea de um quadriltero que tem os lados iguais a 1,4,7 e
8?
Observe inicialmente que 12 82

42 72 e tente mostrar o motivo pelo qual a rea

mxima ser igual a 18 .


Pense na particularidade desses dados e tente deduzir uma condio geral para um
quadriltero tenha um valor mximo para a sua rea.

Soluo do Problema 44
Existe algum natural n tal que n n

( n 1) n seja divisvel por 4027 ?

Usando o fato de que a n b n divisvel por a b para n mpar , podemos verificar


que
nn

( n 1) n ser divisvel por n (n 1) para n mpar .

Note agora que se fizermos n (n 1) =4027 , termos para nossa surpresa n = 2013.

63

Podemos notar que este valor satisfaz s condies do problema, logo n =2013 uma
resposta para o problema.

Soluo do Problema 45
Sejam a,b,c,d nmeros reais positivos. Prove que

1
a

1
b

4 16
c d

64
.
a b c d

Provar esta desigualdade equivalente provar que

1
a

1
b

4 16
(a b c d ) 64 .
c d

(23)

Vamos desenvolver o lado esquerdo da desigualdade (23) :

1
a

1
b

4 16
(a b c d )
c d

4a
4
d

d
4a

2b
2
c

c
2b

22

a
b

4b
4
d

d
4b

Utilizando a Prop 4 , ou seja , x

1
x

b
a

2c
8
d

2a
c

c
2a

d
2c

2 , x 0 para cada termo entre parnteses do

lado direito de (24) , encontraremos :

1
a

1
b

4 16
(a b c d )
c d

22 2 2.2 4.2 2.2 4.2 8.2

Ou seja

1
a

1
b

(24)

4 16
(a b c d ) 64 .
c d

64

Soluo do Problema 46
Escreva 2013 como soma de ( um nmero arbitrrio de) parcelas de modo que o produto das
parcelas seja o maior possvel.

Observe inicialmente que, dado n

N,

se n (n > 4) par, temos


se n (n > 3) mpar, temos

n n
2 2

n 1

n 1

(25)

n.

(26)

Sejam 2013 = n1 + n2 + n3 + nk e
P = n1

n2

n 3 nk

Com as observaes (25) e (26) devemos ter ni


podemos substituir 4 por "2 + 2" e teremos ni
que

= 0; pois se

{ 1, 2, 3, 4} e como 4 = 2

{ 1, 2, 3}; logo P = 1

3 . evidente

= 1, "1 + 2" pode ser substitudo por um 3 e "1 + 3" pode ser substitudo

por "2 + 2". Tambm

2, pois "2 + 2 + 2" pode ser substitudo por "3 +3" ( 3 3 > 2 2

2) e conseqentemente P = 2

3 com ( = 1 ou 2 ). Como 2013 = 3 671 + 0, P = 3671 e S =

3 3 3 ... 3 .
671vezes

(soluo apresentada na revista por Carlos Victor)

Soluo do Problema 47
Represente o nmero 989.1001.1007+320 como um produto de primos.

Faamos x= 991 e seja N = 989.1001.1007+320


N = (x-2)(x+10)(x+16) + 320 = x3+24x2+108x = x(x+6)(x+18)
Logo
N=991.997.1009.
Bastando agora verificar se os valores encontrados so primos. De fato eles so.

65

Soluo do Problema 48
Um quadriltero tem cada um dos seus vrtices sobre os lados de um quadrado de lado
1.Tomando a,b,c, d como sendo os lados deste quadriltero , prove que
2 a 2 b2 c 2 d 2

4 .

Figura 20 ( s Pitgoras )

Pela figura podemos escrever que :

a 2 b2 c 2 d 2
z 2 (1 z )2

x 2 (1 x)2

y 2 (1 y )2

w2 (1 w)2

Agora considere o fato de que :


1
2

x 2 (1 x)2 1

para 0 x1 e em (27) teremos :


2 a 2 b2 c 2 d 2

66

4.

x 2 (1 x)2

(27)

Soluo do Problema 49
Prove que no existe um inteiro tal que a retirada do primeiro dgito produz um resultado
que

1
do inteiro original.
35

Seja N

a1a2 ...ak , onde 0 ai

9 ou seja N

a1.10m

n .

Pelas condies do enunciado :


n

a1.10m n
35

Ou
17 n

a1.2 m 1.5m

que uma igualdade impossvel.

Soluo do Problema 50
Um problema difcil e um resultado interessante : determine o valor de
1 2 1 3 1 4 1 5 ... .

Considere o desenvolvimento para n natural :

n 2

1 4n 4 1 (n 1)(n 3) com n natural .

Podemos escrever

n 2

1 4n 4 1 (n 1)(n 3)

(28)

e como n natural :

(n 2)

1 (n 1)(n 3)

(29)

donde

n(n 2) n. 1 (n 1)(n 3) .
Faamos P(n)

(30)

n(n 2) , logo (30) poder ser escrita da seguinte forma:

P(n) n 1 P(n 1) .
Agora, o mais surpreendente : use a recorrncia (31) para (n+1) e ficamos com

67

(31)

P(n 1) (n 1) 1 P(n 2)

(32)

Logo (31) ficar escrita como


P ( n)

n 1 (n 1) 1 P(n 2)

(33)

ou
P ( n)

n 1 (n 1) 1 (n 2) 1 (n 3) 1 (n 4) ...

Em (34) faa n=1 e tenha a seguinte surpresa


P(1)= 1 2 1 3 1 4 1 5 ...
E como P(n) = n(n+2) ,teremos :
1 2 1 3 1 4 1 5 ... = 3 .

Fascinante, no!

68

(34)

CAPTULO 3: DESCRIO DE LIVROS, REVISTAS E ARTIGOS PARA AS


OLIMPADAS DE MATEMTICA

Neste captulo faremos uma breve anlise de livros que podem ser utilizados para
todos que desejam se aprimorar nas resolues de problemas e tambm para aqueles que de
alguma forma se interessam por matemtica. A escolha foi em funo de abordagens tericas
ou de enunciados dos problemas ou das experincias de professores que preparam alunos para
as diversas Olimpadas de Matemtica. Esclarecemos que a biblioteca aqui registrada no
nica e, possivelmente muitos livros que podem ser referncias ficaro de fora desta e de
alguma forma esses faro partes de futuros trabalhos referentes s Olimpadas de Matemtica.
Importante observar que na dcada de 90, muitos dos livros, revistas ou artigos aqui
registrados no se faziam presentes, mas que foram endossados como enriquecedores para o
nosso atual preparo.
Apesar de muitos livros aqui presentes no serem direcionados para as Olimpadas,
acreditamos que por experincias, sejam no aperfeioamento do professor ou do aluno,
tornam-se inicialmente necessrios para enfrentarmos os problemas dessas competies.

(3.1) BANCO DE QUESTES DA OBMEP

A cada ano a OBMEP vem estimulando professores e alunos


das escolas pblicas a participarem desta Olimpada, que alm de
uma integrao social, fortalece o lao entre o professor de
matemtica e o seu aluno. Oferecida a todas as escolas pblicas, o
Banco de Questes fornece solues das questes anteriores com o
propsito de proporcionar um treinamento para as provas seguintes.
As solues so revisadas e na sua maioria de origem das
apresentadas no stio da OBMEP. As questes divididas por nveis e
temas, Aritmtica, Combinatria e Geometria, so de grande
contribuio para o aluno, de tal forma que a escolha dos problemas
requerem mais raciocnio e criatividade do que conhecimentos
prvios.

Algumas

questes

propostas

na

revista

no

so

necessariamente de provas anteriores, mas apenas para aguar o


69

raciocnio e um treino na criatividade e imaginao do aluno.


(resumo do prprio banco de questes)

(3.2) OLIMPADAS BRASILEIRAS DE MATEMTICA- 1 a 8 ( problemas e resolues)Compilado por lio Mega e Renate Watanabe

Este livro uma coleo dos 110 problemas propostos nas oito primeiras Olimpadas
Brasileiras.
O livro compe-se de trs partes :
Na primeira, encontram-se os enunciados dos problemas classificados por rea e,
dentro de cada rea, por grau de dificuldade. O que difcil para uma pessoa poder ser trivial
para uma outra. Assim, se o leitor encontrar dificuldade em um problema, este fato no
dever impedi-lo de tentar resolver o seguinte .
Na segunda parte do livro, encontram-se solues de todos os problemas. Estas so,
quase sempre, precedidas por um item intitulado Fatos que ajudam e, s vezes, por
Sugestes. Fatos que ajudam so conceitos ou teoremas que permitem resolver o
problema. As sugestes so mais um passo em direo soluo.
Na terceira parte do livro, encontram-se os Apndices, contendo :
1) As 8 provas das Olimpadas ;
2) Uma relao dos alunos premiados nestas Olimpadas;
3) Uma bibliografia.

COMENTRIO :
H um texto no prefcio intitulado Como utilizar o livro .
Este texto para todos aqueles que esto iniciando na arte de resolver problemas
olmpicos que descrevo aqui na ntegra e que se torna til em qualquer outra bibliografia
referente soluo de qualquer problema:
Escolha um problema na rea de sua preferncia leia-o
atentamente, at saber com preciso o que dado e o que pedido .
Tente resolver o problema sem recorrer segunda parte do
livro(solues). Tentar requer pacincia. No desista se a soluo
70

no aparecer na primeira tentativa. uma experincia muito


gratificante encontrar, aps algum tempo, a soluo de um problema
que ,inicialmente, parecia ser muito difcil.
No fique frustrado se no encontrar uma soluo. Recorra, ento,
segunda parte do livro, tomando conhecimento dos fatos que
ajudam e das sugestes. Tente de novo durante algum tempo ... e
s depois estude a soluo apresentada, completando as passagens
no explcitas. A, comece tudo de novo, com outro problema.
Este texto foi assinado pela Comisso de Olimpadas da SBM em So Paulo em
janeiro de 1988.

(3.3) OLIMPADAS BRASILEIRAS DE MATEMTICA (da 9 15) -- LUIZ AMANCIO


MACHADO DE SOUSA JR.- EUFC

Este livro dividido em quatro partes :

Primeira : Enunciados dos Problemas Propostos para as Olimpadas


Segunda : Classificao dos Problemas por Assuntos
Nesta parte o autor divide os problemas propostos em lgebra, Anlise, Combinatria
e Probabilidade, Geometria Analtica, Geometria Plana, Geometria Espacial, Jogos, teoria dos
Nmeros e Outros Tpicos.
Terceira : Solues dos Problemas Propostos
Quarta

: Sugestes e Respostas para os Problemas Propostos. Nesta parte o autor

coloca as sugestes relevantes para a soluo de cada questo.

Alm de uma lista de smbolos utilizados , h um Glossrio dos principais termos e


teoremas usados durante as solues .

COMENTRIOS :
(I) Para uma compreenso total do livro se torna necessrio que conhea alguns tpicos da
matemtica no desenvolvida no Ensino Bsico e cuja bibliografia se encontra no final do
livro .
71

(II) Devido grande experincia do autor em preparo de alunos para as Olimpiadas


Nacional e Internacional, a obra de extrema valia para a formao de professores, para o
aperfeioamento de alunos e para o deleite de todos que gosta de matemtica palavras do
saudoso professor Augusto Csar Morgado .

(3.4) REVISTA EUREKA!

Com a carncia de revistas e livros brasileiros cujos contedos sejam direcionados


para as Olimpadas de Matemtica , a revista Eureka! foi criada com o objetivo de apoiar os
alunos e professores a se desenvolverem neste tipo de competio, com edies impressa e
eletrnica (stio da OBM). Os coordenadores, editores e membros da comisso da revista so
em geral ex-olmpicos e com grandes experincias no preparo de alunos para as olimpadas
Nacionais e Internacionais. A revista atualmente editada duas vezes por ano descritas a
seguir.
1) A primeira revista que geralmente editada no incio do ano, tem como contedos:
a) Os enunciados e solues das trs fases da OBM do ano anterior.
b) Os enunciados e solues das duas fases da OBM universitria do ano anterior.
c) Agenda Olmpica, com as datas das Olimpadas
2) A segunda geralmente editada no final do ano, tem como contedos:
a) Os enunciados e resultados das Olimpadas de Maio, do Cone Sul, Internacional
(IMO), Ibero-Americana, etc....
b) Uma seo de artigos de contedos de Matemtica Elementar divididos como
Avanados, Intermedirios e Iniciantes, complementando contedos do ensino bsico ou
com abordagens mais aprofundadas.
c) Uma seo de Problemas Propostos na qual o leitor convidado a enviar solues
e com as publicaes das melhores posteriormente. Em geral so problemas propostos pela
comisso ou enviados por leitores, cujos enunciados podem ser de outras Olimpadas
Nacionais e Estrangeiras ou at mesmo de livros especializados.
d) Uma seo das Solues dos Problemas Anteriores com caractersticas que visam
o rigor dos conceitos tericos e acrescidos, se necessrio, de comentrios dos editores na
soluo.
72

e) Agenda Olmpica , com as datas das Olimpadas.

(3.5) REVISTA DO PROFESSOR DE MATEMTICA-RPM- Sociedade Brasileira de


matemtica

Uma revista quadrimestral da SBM com apoio da USP amplamente conhecida por
professores de matemtica cujos contedos divididos em sees se tornam um excelente
material de apoio para a sala de aula. Atravs dos seus artigos, ao longo da existncia da
revista, temos vrios temas que so teis no preparo para as Olimpadas de Matemtica. A sua
seo de problemas extremamente interessante, pois so selecionados com a preocupao de
que tenha algum detalhe no muito comum nos livros textos e, muitas das vezes so questes
que fizeram parte de Olimpadas de Matemtica, sejam Nacionais ou Estrangeiras. H
tambm uma seo O Leitor pergunta, na qual uma dvida sobre um problema ou algum
detalhe terico ser devidamente esclarecido pelos responsveis pela seo. Portanto, a
revista um timo material no preparo para as Olimpadas de Matemtica.

(3.6) LGEBRA I A.C.MORGADO; E.WAGNER; M.JORGE -- Livraria Francisco Alves


Editora S.A

Destinado na poca (1974) para alunos do antigo 2 grau, exame supletivo e


vestibulares tornou-se at aproximadamente o ano 2000 uma referncia para os alunos do 9
ano que se preparavam para os concursos do Colgio Naval, Escolas Tcnicas Federais e
Escolas Militares do Exrcito e Aeronutica.
O livro trata inicialmente dos principais conceitos bsicos da aritmtica nos inteiros e
incluindo o estudo inicial de congruncia. Os captulos seguintes so; Potncias, Razes,
Polinmios, Fatorao, Equao do 1 grau, Equao do 2 grau e trinmio do 2 grau. A
teoria e os exerccios (resolvidos e propostos) so boas fontes de consulta para todo aluno do
9 ano que necessita de um preparo inicial para as Olimpadas. O interessante neste livro que
h uma meno sobre quatro problemas (propostos) que at naquele momento ningum tinha
resolvido, inclusive o ltimo Teorema de Fermat. Vale a pena comentar que neste livro
consta a demonstrao que fizemos no problema 18 do captulo 1.
73

Na poca havia comentrios entre professores que teramos o volume 2, mas que
infelizmente no foi editado.

(3.7) GEOMETRIA I -- A.C.MORGADO; E.WAGNER; M.JORGE -- Livraria Francisco Alves


Editora S.A-1974

Um livro com 138 pginas e com a mesma finalidade do livro (3.6), temos uma
referncia em geometria plana Euclidiana com todos os conceitos bem formalizados e
necessrios para um bom desenvolvimento na prtica de resolver problemas de geometria; por
este motivo torna-se um excelente livro de consulta. Neste livro na pgina 111, o exerccio 63
o nosso problema 7 do captulo 1.

(3.8) GEOMETRIA II -- A.C.MORGADO; E.WAGNER; M.JORGE -- Livraria Francisco


Alves Editora S.A-1974

Seguindo a mesma linha do livro (3.7) o livro faz um tratamento complementar da


teoria e inclusive o apndice enriquecido com teorias no presentes nos livros textos
bsicos do ensino fundamental e mdio.

(3.9) TPICOS DE MATEMTICA-IME-ITA-OLIMPADASCARLOS A.GOMES; JOS


MARIA GOMES Livraria VestSeller.

Uma coleo de 6 volumes nos quais cada captulo possui um resumo terico e
questes que foram selecionadas de diversas Olimpadas de Matemtica Nacionais e
Internacionais e de livros especializados. Os autores so experientes no treinamento,
orientando alunos e elaborando materiais didticos para as Olimpadas e para os concursos do
IME ( Instituto Militar de Engenharia) e do ITA ( Instituto Tecnolgico da Aeronutica).
A seguir as divises dos volumes :
Vol 1 : produtos notveis, fatoraes e desigualdades.
Vol 2 : induo matemtica e teoria elementar dos nmeros.
Vol 3 : geometria e trigonometria.
Vol 4 : funes, equaes funcionais, sequncias e sries.
Vol 5 : combinatria e probabilidade.
74

Vol 6 : nmeros complexos, polinmios e equaes algbricas

(3.10) EXERCICES DE GEOMETRIE - - T H. CARONNET- Les Courbes Usuelles

Uma coleo de 9 Livros traduzidos em 1959 por vrios professores brasileiros pela
Editora AO LIVRO TCNICO LTDA - Rio de Janeiro. A coleo, apesar da quase
impossibilidade de encontr-las nas livrarias, tem at hoje uma grande repercusso para os
professores de geometria por conter exerccios com grande grau de dificuldades e, o mais
interessante so as solues que possuem originalidades que dificilmente encontraramos em
outras colees do gnero. Em cada captulo h um resumo terico e enunciados de alguns
teoremas relevantes para as solues. Vale a pena tentar encontrar em sebos ou qualquer outro
companheiro de profisso que possa vender esta coleo. O professor Antonio Luiz Santos
autor do livro Problemas Selecionados de Matemtica [10], possui a coleo completa em
Francs.

(3.11) 104 NUMBER THEORY PROBLEMS -- FROM THE TRAINING OF THE USA IMO
TEAM-TITU ANDREESCU; DORIN ANDRICA; ZUMING FENG- Birkhauser
Uma das referncias para navegar nos mares da teoria dos nmeros. O livro aborda
inicialmente um resumo terico com exemplos de vrias competies internacionais e as
solues apresentadas nos deixam em geral perplexos pelas sacadas e criatividades dos
autores. Costumo geralmente dizer; Grande Titio Andreescu .
(3.12) CONSTRUES GEOMTRICAS E.WAGNER COM A COLABORAO DE
J.P.CARNEIROcoleo do Professor de Matemtica-SBM.

H muito tempo que a disciplina de Construes Geomtricas foi abolida por vrias
escolas, e com isto abriu-se uma lacuna para o crescimento do raciocnio geomtrico de
nossos alunos. Acreditamos que as construes geomtricas fortalecem o nosso raciocnio
para resolvermos problemas de geometria em geral. Este livro resgata juntamente com a
didtica e experincia do autor, a sequncia que devemos ter para uma preciso na construo
de uma determinada figura geomtrica.
O livro est dividido em cinco captulos a saber:
75

Cap 1 --- Construes Geomtricas Elementares;


Cap 2 --- Construes de Expresses Algbricas;
Cap 3 --- reas;
Cap 4 --- Construes aproximadas;
Cap 5 --- Transformaes Geomtricas;
Apndice --- Construes Possveis Usando Rgua e Compasso.
Certamente um livro de uma grande valia para o preparo dos nossos alunos s
competies matemticas.
(3.13) CONSTRUES GEOMTRICASEXERCCIOS E SOLUES SRGIO LIMA
NETO- SBM

Este livro contm todos os exerccios propostos do livro (3.13) e suas solues
didaticamente simples, engenhosas e detalhadas. O Prof. Srgio possui uma publicao das
solues das provas do vestibular do IME (Instituto Militar de Engenharia) dos ltimos 50
anos; um outro trabalho bem difundido para todos que se preparam para este vestibular.
Com as solues elegantes e com figuras bem explicativas, o livro torna-se
indispensvel para os alunos olmpicos2.
(3.14) ELEMENTOS DE MATEMTICA VOL 0 MARCELO RUFINO- Editora e Livraria
VestSeller
O livro aborda do nvel bsico ao avanado os estudos de Potncias e Radiciao,
Bases de Numerao, Mdias, Sistema Mtrico decimal, Razes e Propores, Regra de 3
simples e composta, juros, porcentagens, fraes, dzimas peridicas, equaoes e inequaes
do 1 grau, do 2 grau, biquadradas, modulares, irracionais dentre outros. H uma grande
quantidade de problemas resolvidos e propostos da Epcar, Colgio Naval, e demais
vestibulares nacionais.
Base essencial para os alunos olmpicos.

Os alunos olmpicos aqui denominados, representam todos que de alguma forma se preparam para as Olimpadas da
Matemtica.

76

(3.15) ELEMENTOS DE MATEMTICA VOL 1 MARCELO RUFINO; MRCIO


RODRIGUES Editora e Livraria VestSeller
O livro aborda com profundidade Conjuntos, Funes e Aritmtica (Congruncia,
Equaes Diofantinas, Funo Mximo Inteiro). Esse livro inclui vasta quantidade de
problemas resolvidos e propostos das Escolas Militares, Olimpadas Nacionais e
Internacionais.
Podemos observar pelo o contedo do livro que ele indispensvel para os alunos
olmpicos e tambm para o preparo dos professores.

(3.16) ELEMENTOS DE MATEMTICA VOL 2 MARCELO RUFINO; MRCIO


RODRIGUES Editora e Livraria VestSeller
O livro aborda com excelente profundidade toda a Geometria Plana. Possui vrios
problemas resolvidos e propostos da Epcar, Escola Naval, IME, ITA e Olimpadas Nacionais
e Internacionais.
Podemos observar pelo o contedo do livro que ele indispensvel para os alunos
olmpicos e tambm para o preparo dos professores.
(3.17) ELEMENTOS DE MATEMTICA VOL 3 MARCELO RUFINO; MRCIO
RODRIGUES Editora e Livraria VestSeller
Um excelente livro que aborda as Sequncias, Recorrncia, Combinatria e Matrizes.
Esse livro inclui muitos problemas resolvidos e propostos nas Escolas Militares e Olimpadas
Nacionais e Internacionais.
Certamente de grande valia para os alunos olmpicos.
(3.18) ELEMENTOS DE ARITMTICA A.HEFEZ

O livro foi utilizado no projeto de Mestrado PROFMAT e o interessante que todo


estudo se d no conjunto dos naturais. No captulo de Equaes Diofantinas Lineares h
um conjunto que o autor denomina de Conjuto de Lacunas , uma colocao engenhosa que
em geral no aparece em livros de Teoria dos Nmeros. H neste livro a indicao e uma
discusso do Jogo de Nim.
Certamente de grande valia para os alunos olmpicos.

77

(3.19) TPICOS DE TEORIA DOS NMEROS CARLOS GUSTAVO DE A. MOREIRA;


FABIO E. BROCHERO MARTINEZ;NICOLAU C. SALDANHA- SBM

Um livro escrito por professores ex-olmpicos e medalhistas em diversas Olimpadas


de Matemtica traduz toda a experincia dos autores em preparo de alunos nas Olimpadas
Nacionais e Internacionais.Os autores conseguiram desenvolver os contedos da aritmtica de
uma forma didtica e aprofundada na parte terica e nos exerccios resolvidos.
Uma excelncia no ramo que torna-se necessrio em qualquer biblioteca Olmpica.

(3.20)

THE

SOUTH

AFRICAN

MATHEMATICAL

SOCIETYMATHEMATICAL

OLYMPIAD TRAINING NOTES- MATHEMATICAL TALENT SEARCH- UNIVERSITY OF


CAPE TOWN- JH Webb.
Uma coletnea de Notas no preparo dos alunos que se destacam em Mathematical
Talent Search para a IMO ( Olimpada Internacional de Matemtica), realizada pela
University of Cape Town . As notas so de excelncia em contedos e divididos como a
seguir :
No.1 The Pigeon~hole Principle, by Valentin Goranko
No.2 Topics in Number Theory, by Valentin Goranko
No.3 Inequalities for the Olympiad Enthusiast, by Graeme West
No.4 Graph Theory for the Olympiad Enthusiast, by Graeme West
No.5 Functional Equations for the Olympiad Enthusiast, by Graeme West
No.6 Mathematical Induction for the Olympiad Enthusiast, by david Jacobs
Nesta coleo h um resumo terico com uma profundidade que dificilmente
encontramos em outros artigos do gnero. Essencial para os alunos que esto em um nvel
mais avanado nas Olimpadas, mas que pode ser adaptado para os alunos iniciantes.

78

(3.21) PROOFS WITHOUT WORDS


CLASSROOM RESOURCE MATERIALS / NUMBER / THE MATHEMATICAL
ASSOCIATION OF AMERICA (MAA) - ROGER B. NELSEN MAA-1993
Proofs without words como o prpio titulo descreve, so figuras ou diagramas de
resultados matemticos que foram editados regularmente em revistas e artigos pela MAA, e
cujas demonstraes so elaboradas apenas por aspectos puramente geomtricos visuais .
As figuras instigam o leitor a pensar geometricamente sobre a igualdade ou a
desigualdade proposta . Possui sees divididas nos seguintes assuntos :
1) Geometria e lgebra
2) Trigonometria, Clculo e Geometria Analtica.
3) Desigualdades.
4) Sequncias e Sries.
5) Uma miscelnea de resultados que nos deixam pasmos.
Este livro se torna extremamente valioso e eficaz para aguar o raciocnio e a
criatividade do aluno para as Olimpadas. Todo jovem olmpico deve ter este livro.

(3.22) POWER PLAYEDWARD J. BARBEAU- MAA1997


O autor faz uma referncia no prefcio do livro sobre o nmero 142857, o qual tem a
propriedade de que seus seis primeiros mltiplos envolvem os mesmos seis dgitos numa
mesma ordem cclica.
Os captulos so descritos da seguinte forma :
Cap 1 :Inteiros mpares e quadrados.
Cap 2 : Triplas Pitagricas e relativos.
Cap 3 : Sequncias.
Cap 4 : Equao de Pell.
Cap 5 : Somas iguais de iguais potncias.
Cap 6 : Dgitos e somas de sotncias
Cap 7 : Conjuntos interessantes .
Indispensvel esse livro para desenvolvermos a nossa capacidade de potncias.

79

CONSIDERAES FINAIS

Considerando como um foco de estudo o impacto para um professor de matemtica


sem experincia em Olimpadas nesta disciplina, decidimos analisar estratgias de
aperfeioamento profissional com a finalidade de que ele enfrente os desafios propostos nas
questes dessas atividades e que ele desenvolva um conjunto de aes na resoluo dos
problemas das Olimpadas de Matemtica.
Temos, anualmente, o crescimento da quantidade de alunos participantes de tais
competies, principalmente na OBMEP, devido sua divulgao intensiva nas escolas
pblicas. O que devemos ressaltar a forma como se d esse crescimento, ou seja, se h,
simplesmente, imposio para que uma determinada turma realize as provas ou se existem
estmulos mostrando a importncia da referida competio.
Para aqueles alunos que possuem afinidades com a Matemtica, a Olimpada torna-se
um medidor para o raciocnio e a sua criatividade. Mesmo assim, a prova pode se tornar um
fator decepcionante se um preparo prvio no ocorrer, seja atravs das provas anteriores ou
atravs de grupos de estudo com material especializado. Para os alunos que por algum motivo
se sintam desinteressados em estudar Matemtica, realizar a competio se torna um martrio
e, provavelmente, um acrscimo nesse desinteresse, j que esto realizando a prova por uma
imposio.
Fazer o aluno gostar de estudar Matemtica uma tarefa desafiadora para o professor,
e criar instrumentos incentivadores para esse jovem competir numa Olimpada de Matemtica
deve ser uma preocupao inserida nessa tarefa.
Acreditamos que atravs de projetos internos na escola, juntamente com o grupo de
orientadores, os diretores e uma biblioteca especfica, o professor conseguir bons resultados
para a escola nessas Olimpadas. Para isso o incentivo no uso da tecnologia fornecido nos
sitos na internet das Olimpadas um instrumento que ajuda no aprendizado ([1], pag 80).
Para professores que de alguma forma, no passado, participaram mais intensamente de
Olimpadas de Matemtica como alunos, os instrumentos incentivadores e os pr-requisitos
necessrios para um bom desempenho dos jovens nas Olimpadas so ferramentas que ele
sabe onde encontrar. Podemos tomar um instrumento incentivador na escola, como sendo
algum tipo de premiao de todo o corpo pedaggico e, como pr-requisito uma biblioteca
destinada a esse tipo e competio, por exemplo.

80

O que tratamos ao longo de todo esse trabalho foi uma preocupao que em geral est
presente nos professores mesmo entre os experientes no preparo de alunos para diversos
concursos, isso , como dirimir as dificuldades presentes nas Olimpadas de Matemtica?
Afirmar que a soluo foi descrita nesse trabalho, uma revelao muito prepotente,
mas temos cincia de que a prtica de resolver problemas, juntamente com uma biblioteca
direcionada, certamente propicia uma minimizao nas dificuldades, e cada vez mais,
estaremos capacitados a preparar e a incentivar os alunos para a competio e, como
resultado, os objetivos dos organizadores das Olimpadas sero atingidos.
Podemos nesse momento, atentar para uma parte do ttulo do nosso trabalho; Que
preciosidades envolvem os problemas desta competio? De acordo com os problemas aqui
apresentados, observamos como determinados detalhes nos fomentam a passear no mundo da
criatividade de suas resolues e, juntamente com a imaginao do autor que props a
questo, no difcil de responder pergunta.
O que nos levou a introduzir a segunda pergunta do ttulo deste trabalho foi uma
vivncia com professores que de alguma forma tinham e tm receios principalmente nas fases
finais dessas Olimpadas. Talvez possamos conjecturar as possveis dificuldades existentes:
uma falta de exigncia externa, ou seja, escolas que de alguma forma no tenham um
compromisso de preparar o aluno para as competies; ou por dificuldades presentes nas
questes e cujo motivo da existncia era desconhecido. Essas conjecturas podem, talvez,
explicar as possveis resistncias de muitos serem temerosos no que se refere s Olimpadas
de Matemtica.
O professor pode ser excelente na atuao de suas atividades escolares dentro da grade
curricular de Matemtica do Ensino Bsico e, no entanto, no possuir facilidade nas
resolues dos problemas propostos nas Olimpadas. Fica, ento, claro o impacto para o
docente de Matemtica sobre o que essas desafiadoras questes podem lhe trazer.
Estamos cientes que toda a trajetria seguida nesse trabalho pode no ser nica, mas
visamos alcanar aqueles que por algum motivo no tiveram a oportunidade de participar,
como ex-olmpicos ou como agentes preparadores de alunos nesse tipo de competio, e
visando, ainda, a todos que desejam encantar-se com as prolas matemticas envolvidas nas
questes olmpicas. Dessa forma, estaremos, tambm, contribuindo com um desejo do
Ministrio da Educao, que a melhoria do ensino da Matemtica no Brasil.
A cada ano, os projetos de Olimpadas se intensificam, haja vista o crescimento de
outras Olimpadas, como as de Informtica, Fsica, Lngua Portuguesa. Com esse tipo de
81

avano, as formas de interao entre o preparo de alunos e professores tomar rumos que
talvez sejam diferentes daqueles descritos nos diversos trabalhos sobre as Olimpadas,
inclusive os descritos aqui. Tais consequncias podero vir a ser tema de novos trabalhos que
futuramente sero alvo de outros pesquisadores sobre a Olimpada de Matemtica ou sobre
outras Olimpadas.
Por fim, a motivao desse trabalho deve-se a uma experincia prpria e forma como
ocorreu o decrescimento dos incmodos nessa deslumbrante competio, o que est
explicitado ao longo dos captulos 2 e 3. Muitos desses problemas desenvolvidos da forma
aqui descrita, integraram as aulas na preparao de alunos, em escolas pblicas, municipais e
federais, cujos resultados foram tambm os incentivadores para que essa dissertao tenha
sido realizada.
Apesar da vontade de construir um trabalho semelhante a este, surgir anteriormente, o
PROFMAT abriu as portas para que o sonho fosse realizado. Esperamos que novas produes
voltadas ao mesmo tema sejam frutos de novas dissertaes ou livros, complementares a esse
trabalho, inteno essa vislumbrada para ocorrer em futuro prximo. A proposta deste
trabalho foi incentivar a todos aqueles que por algum motivo se sentem inseguros nos
problemas olmpicos e no sabem como lapidar esta prola que a Olimpada de
Matemtica.

82

REFERNCIAS

[1] ALVES, Washington,J.S.; O Impacto da Olimpada de Matemtica em Alunos da Escola


Pblica; PUC/SP-2010 ( Dissertao de Mestrado Profissional Em Ensino de Matemtica);
[2] ANDREESCU,T., FENG, Z.; Mathematical Olympiads 1998-1999,problems and
solutions from Around the world; MAA, 2000;
[3] ANDREESCU, T.; Old and New Inequalities ; MAA-2005;
[4] BIONDI, R. L.; VASCONCELOS, L. e MENEZES FILHO, N. A. de: Avaliando o
impacto da Olimpada Brasileira de Matemtica das Escolas Pblicas (OBMEP) no
desempenho de matemtica nas avaliaes educacionais. Disponvel em :
http://bibliotecadigital.fgv.br/ocs/index.php/sbe/EBE09/paper/view/1092/315.

Acesso

em

12/02/2013.
[5] da Comisso de Olimpadas da Sociedade Brasileira de Matemtica (SBM); Olimpadas
Brasileiras de Matemtica 1 a 8(problemas e resolues)-compilado por lio Mega e
Renate Watanabe. Atual Editora Ltda, So Paulo,1995;
[6] FUNCTION MAGAZINE; A SCHOOL MATHEMATICS JOURNAL- MONASH
UNIVERSITY-2003;
[7] HONSBERGER,R; Mathematical Morsels; Mathematical Association of American
(MAA), Whashington,1978;
[8] HONSBERGER,R; More Mathematical Morsels; Mathematical Association of American,
Whashington,1991;
[ 9] LARSON, L. C.;Problem-Solving Through Problems; Springer-Verlag; 1983
[10] MARCOS, V. M.M.-CMPA; B. ,MARCUS,V.A.-UFRGS; Olimpada Brasileira de
Matemtica das Escolas Pblicas (OBMEP): As Origens de um Projeto de Qualificao do
ensino de matemtica na Educao Bsica-X Encontro Gacho de Educao Matemtica2009/Iju/RS
[11] MOREIRA, C. G. T. DE A.; WAGNER, E.. 10 Olimpadas Iberoamericanas de
Matemtica. OEI,1996;
[12] NASCIMENTO, M. G.; OEIRAS, J. T.Y.; Olmpico: Um ambiente Virtual para
Competies Escolares Via Internet.Belm, PA:UFPa, 2006;
83

[13] PERAINO, M. A.; Adolescente Com altas habilidades/Superdotao de um


assentamento rural: um estudo de caso : campo Grande, Mato Grosso do Sul:2007.104f
(Dissertao de Mestrado em Psicologia). UCDB;
[14] REVISTA EUREKA!; Olimpada Brasileira de Matemtica-SBM-IMPA;
[15] SANTOS, A.L.; Problemas Selecionados de Matemtica; Editora Cincia Moderna
Ltda,Rio de Janeiro,2006;
[16] SHKLARSLY,D.O.;The USSR Olympiad Problem Book: selected problems and
theorems of elementary mathematics; Dover Publications, Inc., New York,1993;
[17] SOIFER,A.;Colorado Mathematical Olympiad: The First ten Years and Further
Explorations;Springer;U.S ( Paperback-june-1994);
[18] SOUSA JR,L.A.M.; Olimpadas Brasileiras de Matemtica da 9 15 Edies
UFC,Fortaleza,1994;
[19] SUCUPIRA, G..;Ser que as Meninas No gostam de Matemtica? Reflexo sobre
Gnero, Educao e Cincia a partir de uma Etnologia sobre as Olimpadas de matemtica
em Santa Catarina. SC : UFSC-2008;

84

APNDICES
APNDICE A : SEMANA OLMPICA

Para os alunos premiados na Olimpada Brasileira de Matemtica, vem sendo


realizada desde 1998, um treinamento durante uma semana por uma equipe de professores
especializados em preparo de alunos para as Olimpadas. Esses jovens representaro o Brasil
nas competies internacionais de Matemtica e alm de proporcionar aos alunos um
intensivo preparo, os jovens tm momentos oportunos de uma nova integrao com outros
alunos da mesma faixa etria e com interesses similares. Este ano a XVI Semana Olmpica
aconteceu em Aracaju com apresena dos alunos premiados na OBM e Universitria.
Os contedos discutidos foram :
1) Combinatria Geomtrica
2) Contagem
3) At que tamanho podemos brincar de esconde-esconde ?
4) lgebra para Intermedirios-mximos e mnimos e outras ideias teis
5) Bissetrizes e suas propriedades
6) Por que o quadrado de terminados em 5 to fcil?
7) Como resolver problemas difceis usando Bhaskara
8) Como cobrir tabuleiros
9) Teoria aditiva dos nmeros
10) Nmeros primos e seus mistrios
11) geometria
Todo o material da Semana Olmpica, inclusive os anteriores podem ser encontrados no sito
na internet da OBM.

85

APNDICE B : RETA DE SIMSON

( Retirado de um artigo escrito por Marcelo Mendes de Oliveira )


Reta de Simson
Nvel 2
Marcelo Mendes de Oliveira
marcelom@ceara.net
Introduo

Apesar de essa reta famosa ter sido descoberta por William Wallace em 1797, por
descuido atribuiu-se falsamente, poca, o resultado a Robert Simson (1687-1768). Os
seguintes problemas apresentam propriedades e aplicaes da Reta de Simson ou Reta de
Simson-Wallace.
Reta de Simson
RETA DE SIMSON: Se perpendiculares so traadas a partir de um ponto sobre o circuncrculo
de um tringulo a seus lados, suas intersees com os lados do tringulo so colineares e
pertencem Reta de Simson. (A recproca tambm verdadeira.)
M
X

A
LL

C
P

De fato,
disso,

NLC =
NXC e

NXC (quadriltero NLXC) e


AXM somados com

ALM =

AXN suplementam

AXM (quadriltero ALXM) e, alm


ABC. Segue que

NLC =

ALM

e M, L, N so colineares.
NOTA: Se a perpendicular XN a BC corta o circuncrculo novamente em P, ento AP
paralela Reta de Simson de X.

86

APNDICE C : POTI - POLOS OLMPICOS DE TREINAMENTO INTENSIVO

A partir de 2012 a OBM, OBMEP e o IMPA iniciaram um projeto de


treinamento para os alunos com aulas presenciais em plos especificados. H no sitio do
POLO

material

didtico

aulas

em

vdeo,

como

descreve

prprio

sitio

(http://poti.impa.br/index.php/site/) :
1)Sobre o POTI
A Olimpada Brasileira de Matemtica das Escolas Pblicas (OBMEP), Olimpada
Brasileira de Matemtica (OBM) e o Instituto Nacional de Matemtica Pura e Aplicada
(IMPA) esto dando continuidade em 2013 ao programa Polos Olmpicos de Treinamento
Intensivo (POTI) onde sero oferecidos, ao longo de todo o ano, cursos gratuitos de
matemtica para os estudantes de todo o Brasil.
O programa destinado aos interessados em participar da OBM e ou OBMEP e que
estejam matriculados no oitavo ou nono anos do Ensino Fundamental (nvel II) ou em
qualquer uma das sries do Ensino Mdio (nvel III).
O curso para cada um dos nveis cobrir os contedos de lgebra, Combinatria,
Geometria Plana e Teoria dos Nmeros. O site oficial disponibiliza todo material terico e os
vdeos das aulas correspondentes.
2) Para os alunos que no tiverem acesso aos polos presenciais, o POTI disponibiliza
aulas em vdeo gravadas no Instituto Nacional da Matemtica Pura e Aplicada (IMPA) por
uma equipe de renomados professores que contam com ampla experincia de treinamentos
para alunos de competies nacionais e internacionais.
Para um melhor aproveitamento das aulas em vdeo, recomendamos que os alunos e
professores faam uso do material terico correspondente.
3) O material didtico do POTI contempla toda a matemtica olmpica das sries
finais do ensino fundamental e de todo o ensino mdio. Todo material didtico produzido
pelo POTI que se encontra disponvel nesta pgina e pode ser usado e distribudo livremente
para quaisquer finalidades no comerciais.
Cada aula um arquivo contendo teoria, problemas e exerccios propostos visa
estimular o estudo autodidata de alunos que no possuam acesso Polos presenciais

87

APNDICE D : NGULOS ADVENTCIOS

Com relao figura , dados os ngulos a,b,c mltiplos de 1, com b > c. A


tripla (a,b,c) Adventca se o ngulo correspondente x tambm um mltiplo de 1. O
ngulo x denominado de ngulo Derivado.

Retirado do artigo "Adventitious angles", Colin Tripp,Mathematical Gazette, 59, pp 98-106,


1975.

88

APNDICE E : UMA SOLUO TRIGONOMTRICA PARA O PROBLEMA 7

Usando a Lei dos senos nos tringulos PQB, BCQ e PQC , teremos :

BQ
PQ
sen( x 40) sen(30 )
BQ
BC
sen(80 ) sen(50 )
CQ
PQ
sen( x) sen(20 )

Eliminando BQ, PQ, BC, CQ com BC = QC , encontraremos :

Senx.sen(80) = 2sen(x+40).sen(50).sen(20) ; ou seja


Senx = sen(x+40).[ sen(130) sen(30)] ; ou
Senx = sen(x+80) +senx sen(x+40) ; onde devemos ter :

x + 80 + x + 40 = 180 , o que acarreta x = 30 .

89

APNDICE F : UMA SOLUO TRIGONOMTRICA PARA O PROBLEMA 21

Seja

OBA

sen(2) = BC/2BO
AP/sen(2) = AO/sen(3)
Logo
2 sen(2) = sen(2)/ sen(3)
sen(3) = 1/2
= 30 .

Um Comentrio: 3 pode ser igual a 150 ?

90

Vous aimerez peut-être aussi